NOTEPAD
Results
of 200 questions answered correctly

You have reached of 200 points, ( %)

Your time

Question 1 of 200

1. A patient diagnosed with focal tuberculosis of the upper lobe of the right lung had been taking isoniazid as a part of combination therapy. After some time, the patient reported of muscle weakness, decreased skin sensitivity, blurred vision, impaired motor coordination. Which vitamin preparation should be used to address these phenomena?

Explanation

Isoniazid is the hydrazide of isonicotinic acid and is a pyridine. Pyridine occurs in many important compounds including azines and the Vitamins Niacin(B3) and Pyridoxine(B6). Therefore, isoniazid can be interfered with Vit.B3,B6 and even B1 metabolism by competing with them.

Isoniazid is a first line antituberculosis drug that inhibits the synthesis of mycolic acid. Vit B6 is needed for the transformation of tryptophan to Vit.B3.

 

Adverse reaction of Isoniazid: peripheral neuritis, optic neuritis, hepatitis and idiosyncratic hepatotoxicity.

2. A 60-year-old male patient has a 9-year history of diabetes and takes insulin Semilente for the correction of hyperglycemia.10 days ago he began taking anaprilin for hypertension. One hour after administration of the antihypertensive drug the patient developed hypoglycemic coma. What is the mechanism of hypoglycemia in case of anaprilin use?

Explanation

krushkrok No196 (2014)

Anaprilin is a beta-blocker. It will inhibit epinephrine effect together with glucagon. Epinephrine normally breaks down glycogen (glycogenolysis) to glucose and this makes glycogen available for use in the body. Since Anaprilin inhibits epinephrine effect,it therefore inhibits glycogenolysis. Insulin also is a strong inhibitor of glycogenolysis. Therefore, the combination of both Insulin and Anaprilin can lead to hypoglycemic coma.

3. Pterin derivatives (aminopterin and methotrexate) are the inhibitors of di- hydrofolate reductase, so that they inhibit the regeneration of tetrahydrofolic acid from dihydrofolate. These drugs inhibit the intermolecular tranfer of monocarbon groups, thus suppressing the synthesis of the following polymer:

Explanation

IMG_9914

Pterin derivatives (aminopterin & methotrexate) are folic acid analogs that inhibit the reduction of dihydrofolate to tetrahydrofolate, catalyzed by dihydrofolate reductase. These drugs limit the amount of tetrahydrofolate available for use in purine synthesis & this slows down DNA replication (reduction in dTMP) in mammalian cells. These compounds are therefore,useful in treating rapidly growing cancers,but are also toxic to all dividing cells.

4. A child with suspected tuberculosis was given Mantoux test. After 24 hours the site of the allergen injection got swollen, hyperemic and painful. What are the main components that determine such response of the body?

Explanation

FullSizeRender (7)

Mantoux test is a type IV Hypersensitivity reaction (HSR), which involves macrophages,T-lymphocytes and lymphokines(cytokines). Mononuclear cells (lymphocytes,monocytes,macrophages).

Remember,it is antibody independent (i.e does not involve antibodies).

 

B-lymphocytesPlasma cellsIg(Antibodies)------- none is involved in Type IV HSR.

5. Hemoglobin catabolism results in release of iron which is transported to the bone marrow by a certain transfer protein and used again for the synthesis of hemoglobin. Specify this transfer protein:

Explanation

Transferrin is the binding protein of iron and is synthesized in the liver. Iron bound to transferrin is the serum iron. Absorbed Iron is either stored in the cytoplasm as mucosal ferritin or binds to plasma transferrin and transported to the erythroid precursors in the bone marrow. 

6. A 12-year-old boy has been hospitalized for suspected food poisoning. The fecal samples were inoculated on the Endo agar, which resulted in growth of a large number of colorless colonies. What microorganism is most likely to be EXCLUDED from the list of possible causative agents of the disease?

Explanation

Both Endo Agar and EMB agar (Eosin-Methylene-Blue-Agar) are selective stain for Gram–ve bacteria, growth of gram+ve organisms is inhibited. They provide a color indicator distinguishing between organisms that ferment lactose (e.g E.coli) and those that do not (e.g salmonella). Lactose fermenters (e.g E.coli) give a green metallic sheen. Lactose non fermenters produce clear, colourless colonies. All other options left are lactose non fermenters. Only E.coli is a lactose fermenter and therefore cannot be colourless.

7. A 23-year-old patient has been admitted to a hospital with a craniocerebral injury. The patient is in a grave condition. Respiration is characterized by prolonged convulsive inspiration followed by a short expiration. What kind of respiration is it typical for?

Explanation

APNEUSTIC: Prolonged, gasping inspiration followed by an insufficient or brief expiration. E.g damage to brainstem, ketamine.

KUSSMAUL: Vigorous inhalation and active exhalation with forced contraction of expiratory muscles. E.g in diabetic ketoacidosis.

CHEYNE-STOKES: Cyclic breathing pattern which regular change in frequency (at time faster & followed by a gradual decrease) that results in a temporary stop in breathing (apnea). E.g heart failures,strokes.

BIOT: periods of apnea interchanging with periods of resumption of breathing without significant changes in the amplitude of respiratory movements. E.g damage to pons due to stroke or trauma.

GASPING: forced,convulsive breath.

8. It has been experimentally proven that the excitation of the motor neurons of flexor muscles is accompanied by the inhibition of the motor neurons of extensor muscles. What type of inhibition underlies this phenomenon?

Explanation

krushkrok No8 (2013)

Reciprocal inhibition is the inhibition of antagonistic muscles when a group of muscles are activated, an important feature of both flexor and extensor reflexes, as a result of reciprocal innervation. Usually, excitation of one group of muscles is associated with the inhibition of another…i.e antagonistic group of muscles on the same side. E.g when a flexor reflex is elicited, the flexor muscles are excited (contracted) and the extensor muscles are inhibited (relaxed) in that side.

9. A 3-year-old boy with pronounced hemorrhagic syndrome doesn’t have anti-hemophilic globulin A (factor VIII) in the blood plasma. Hemostasis has been impaired at the following stage:

Explanation

In the intrinsic (internal) coagulation system: Four component complex is formed (Factor VIII, IXa, Phospholipid and Calcium)- this complex activates Factor X in the final common pathway.

FullSizeRender

10. A patient got a gunshot wound of hip which damaged the sciatic nerve. Any impact on the affected limb causes severe, excruciating pain. What mechanism of pain is most likely in this case?

Explanation

Causalgia is a rare pain syndrome related to partial peripheral nerve injuries. The peripheral nervous system encompasses nerves that extend from the CNS of the brain & spinal cord to serve limb & organs. The gunshot wound damged a peripheral nerves (sciatic nerve) from spinal cord supplying the lower limb.

 Phantom pain: pain from non-existent body structures (amputated limbs).

 

Phantom – if a leg has been amputated, the cut end heals with scar formation. The cut ends of nerve fibers are merged within the scar. If the cut end of sensory fibers are stimulated during movement of thigh, the patient feels as if the sensation is originating from non-existent leg. Sometimes the patient feels pain in non-existent limb.

11. A 60-year-old patient with a long history of stenocardia takes coronarodilator agents. He has also been administered acetylsalicylic acid to reduce platelet aggregation. What is the mechanism of antiplatelet action of acetylsalicylic acid?

Explanation

FullSizeRender (1)

Acetylsalicyclic acid (Aspirin) is an NSAID. Cycloxygenase, the enzyme which converts arachidonic acid into the endoperoxide precursors of prostaglandin, has at least two different isoforms: COX-1 & COX-2. COX-1 is primarily expressed in non-inflammatory cells whereas COX-2 is expressed in activated lymphocytes, polymorphonuclear cells and other inflammatory cells. Acetylsalicyclic acid and the older non-selective NSAIDs inhibit both cyclooxygenase isoforms & thereby decrease prostaglandin & thromboxane synthesis (disaggregating effect) throughout the body.

 
12. A patient with bronchial asthma has developed acute respiratory failure. What kind of respiratory failure occurs in this case?

Explanation

krushkrok No50 (2009)

Obstructive respiratory disease is the abnormal respiratory condition characterized by difficulty in expiration. E.g asthma,chronic bronchitis, emphysema, cystic fibrosis. Restrictive respiratory disease is the abnormal respiratory condition characterized by difficulty in inspiration. E.g poliomyelitis, myasthenia gravis, paralysis of diaphragm, spiral cord diseases, pleural effusion, fibrosis. (lung fibrosis-pneumofibrosis).

Forced expiratory volume (FEV) is the volume of air, which can be expired forcefully in a given unit of time. It is very much decreased in obstructive diseases like asthma and emphysema. 

13. On the fifth day after the acute blood loss a patient has been diagnosed with hypochromic anemia. What is the main mechanism of hypochromia development?

Explanation

Acute hemorrhage refers to sudden loss of a large quantity of blood as in the case of accident. Within about 24 hours after the hemorrhage, the plasma portion of blood is replaced. However the replacement of RBCs does not occur quickly and it takes at least 4-6 weeks. On the 5-7th day (regenerative or bone marrow period). There is appearance of some immature forms of RBC (i.e. normoblast, reticulocyte) with decreased hemoglobin and colour index (hypochromia).

14. A patient with diabetes developed a diabetic coma due to the acid-base imbalance. Specify the kind of this imbalance:

Explanation

During insulin deficiency, glucose cannot be utilized by the peripheral tissues for energy. So, a large amount of fat is broken down to release energy. It causes the formation of excess ketone bodies leading to acidosis. One more reason for acidosis is that the ketone bodies are excreted in combination with sodium ions through urine (ketonuria). Sodium exchanged H+, which diffuse from the renal tubules into ECF adding to acidosis.  In diabetic patients, the body produces too much acid & the kidneys are not removing them fast enough as they are produced which results in metabolic acidosis.

15. A girl receives antibiotics of the penicillin group for acute bronchitis. On the third day of treatment she developed allergic dermatitis. Which drug should be administered?

Explanation

Loratadine : Antihistamine (antiallergic) block H1-histamine receptors as well as adrenergic, cholinergic & serotonin-binding recpetors. They do not influence the formation or release of histamine, but rather they competitively block the receptor-mediated response of target tissue.

Cromolyn Sodium: Membrane stabilizers , anti-allergic but it’s best for prophylaxis (prevention) & not for treatment of allergic conditions.

Beclomethasone: Synthetic glucocorticoids and anti-inflammatory.

Ephedrine hydrochloride: Adrenomimetic.

Levamisole: Synthetic immunostimulator.

16. A female patient has been diagnosed with cervical erosion, which is a precancerous pathology. What defense mechanism can prevent the development of a tumor?

Explanation

NK cells are lymphocytes that are capable of destroying tumor cells without prior sensitization and thus may provide the 1st line of defense against tumor cells. After activation with IL-2 and IL-15, NK cells can lyse a wide range of human tumors, including many that seem to be non-immunogenic for T-cells. NK cells, T-cells & macrophages may collaborate in antitumor reactivity.

17. Microscopy of the coronary artery of a dead 53-year-old patient revealed luminal occlusion due to a fibrous plaque with some lipids. The most likely form of atherosclerosis in this case is:

Explanation

Stages of atherosclerosis:

Pre-lipid stage: mucoid swelling of intima, destruction of endothelium, elastic & collagen fibres of intima’s basal membrane.

Stage of fatty stripes (lipidosis): Fatty stripes appear on intima. Macrophages that have accumulated lipid in their cytoplasm appear as csantomic or foam cells.

Stage of liposclerosis: Lipid accumulates not only in macrophages but also in smooth muscle cells. In areas of lipidosis (lipid core), a young CT grows & forms a fibrous cap/plaque.

Stage of atheromatosis: Necrosis of central part of fibrous cap, with formation of atheromas. Atheromas consist of amorphous lipid-rich material & are soft.

Stage of ulceration: Break of fibrous cap cover & ulceration with small hemorrhage into plaque.

Stage of atherocalcinosis: Deposition of calcium in ulcerative plaque. Calcification of vessels leads to hardening of arteries, which appears white & petrosal.

18. Autopsy of the patient revealed bone marrow hyperplasia of tubular and flat bones (pyoid marrow), splenomegaly (6 kg) and hepatomegaly (5 kg), enlargement of all lymph node groups. What disease are the identified changes typical for?

Explanation

In chronic leukemia: spleen can weigh (6-8kg); liver (5-6kg). Chronic myelogenous leukemia (CML) is characterized by hepatosplenomegaly and generalized painless lymphadenopathy; pyoid bone marrow.

19. As a result of an injury a patient cannot extend his arm at the elbow. This may cause abnormal functioning of the following muscle:

Explanation

krushkrok No29a (2014) Triceps brachii is the chief extensor of the forearm at the humeroulnar joint. Has a long, lateral, and medial head. Inserts on olecranon of ulna bone. Infraspinatus muscle: Lateral (external) rotation of the humerus at the glenohumeral joint, also assists in holding the head of the humerus in the glenoid fossa. Teres Major muscle: Adduction & medial rotation of the humerus at the glenohumeral joint shoulder joint) Subscapularis muscle: Medial (internal ) rotation and adduction of the humerus at the glenohumeral joint, also assists in holding the head of the humerus in the glenoid fossa. Levator scapulae muscle: Elevates the scapula.
20. A man sitting with his eyes closed, undergoes electroencephalography. What rhythm will be recorded on the EEG if there is an audible signal?

Explanation

Beta rhythm???
21. Electrophoretic study of a blood serum sample, taken from the patient with pneumonia, revealed an increase in one of the protein fractions. Specify this fraction:

Explanation

Electrophoretic study of a blood serum sample examines specific proteins in the blood called GLOBULINS. With five major fractions: -Serum Albumin, Alpha-1 Globulin, Alpha-2 Globulin, Beta-Globulin, Gamma-Globulin.

Alpha-2 Globulin is INCREASED in nephrotic syndrome.

Beta-Globulin is INCREASED in Iron Deficiency Anaemia.

Gamma-Globulin is INCREASED in Severe Infection (pneumonia), chronic liver disease. Due to Ig produced against the infection.

22. Examination of an 18-year-old girl revealed the following features: hypoplasia of the ovaries, broad shoulders, narrow pelvis, shortening of the lower extremities, \"sphinx neck\". Mental development is normal. The girl was diagnosed with Turner’s syndrome. What kind of chromosome abnormality is it?

Explanation

krushkrok No105 (2014)FullSizeRender (1)These characterize Turner’s syndrome (XO). It can be complete monosomy (45,XO) or mosaicism (e.g. 45,XO/46,XX).
23. Hypertrichosis is the Y-linked character. The father has hypertrichosis, and the mother is healthy. In this family, the probability of having a child with hypertrichosis is:

Explanation

Hypertrichosis is Y-linked, therefore all the male child will have this pathology. The probability of having a male child is 50% (0.5). Since,all the male child of this man have the pathology,it therefore means that the probability of inheriting this pathology (hypertrichosis) is 50% or 0.5

24. A casualty has a fracture in the region of the inner surface of the left ankle. What is the most likely site for the fracture?

Explanation

krushkrok No24 (2013)

The ankle is where the leg and foot meets (Talocrural joint). This joint is formed by the tibia (medially), fibula (laterally) and talus. Inner/medial surface of the ankle is formed by the medial malleolus of tibia. The inferomedial process of the tibia forms the medial malleolus. The inferior process of the fibula forms the lateral malleolus.

 

Lower third of the fibula is located laterally. Calcaneus is a tarsal bone and is not involved in forming the ankle joint.

25. Some infectious diseases caused by bacteria are treated with sulfanilamides which block the synthesis of bacteria growth factor. What is the mechanism of their action?

Explanation

IMG_9914

Sulfonamides inhibit folate synthesis. They also act as antimetabolites of para-amino benzoic acid PABA by inhibiting dihydropteroate synthase.

26. A 42-year-old male patient with gout has an increased blood uric acid concentration. In order to reduce the level of uric acid the doctor administered him allopurinol. Allopurinol is the competitive inhibitor of the following enzyme:

Explanation

\"image\"Allopurinol is a purine analog. It reduces the production of uric acid by competitively inhibiting the last two steps in uric acid biosynthesis that are catalyzed by xanthine oxidase. NOTE: Uric acid is less water soluble than its precursors. When xanthine oxidase is inhibited, the circulating purine derivatives (xanthine & hypoxanthine) are more soluble and therefore are less likely to precipitate.

 The end product of the purine nucleotides catabolism in humans and other primates is uric acid (urate) which is excreted in urine. Allopurinol and febuxostat inhibits Xanthine oxidase (XO). Hypoxanthine and Xanthine which is more soluble is excreted in urine. Purine nucleotides (adenine and guanine). AMP – Adenosine monophosphate; GMP – Guanosine monophosphate

27. A 40-year-old female patient diagnosed with acute pancreatitis has been delivered to the admission department of a regional hospital. What drug should be administered the patient in the first place?

Explanation

Acute pancreatitis is an inflammatory disease characterized by the autodigestion of pancreas by pancreatic enzymes (proteolytic enzymes). Contrycal (Aprotinin, Gordox)- protease inhibitor. Therefore,it can inhibit the proteolytic enzymes digesting the pancreas.

Platyphyllin hydrotartras, Metacin, Pirenzepine, Atropine Sulfate – M-cholinoblockers. Contrycal also has antifibrinolytic (antiplasmin) and hemostatic effect.

28. A patient consulted a doctor about being unable to abduct his right arm after a past trauma. Examination revealed that the passive movements were not limited. The patient was found to have the atrophy of the deltoid muscle. What nerve is damaged?

Explanation

krushkrok No28 (2013)

Deltoid muscle:

Origin: lateral third of clavicle, acromion of scapula, spine of scapula.

Insertion: Deltoid tuberousity of humerus.

Innervation: Axillary nerve ( a terminal branch of branchial plexus receiving fibres from C5 & C6 ventral rami).

Action:

*Anterior part- flexion & medial (internal) rotation of the humerus at glenohumeral joint.

*Middle part- abduction of the humerus at the glenohumeral joint.

*Posterior part – extension & lateral (external) rotation of the humerus at the glenohumeral joint.

29. After a trauma of the upper third of the anterior forearm a patient exhibits difficult pronation, weakening of palmar flexor muscles and impaired skin sensitivity of 1-3 fingers. Which nerve has been damaged?

Explanation

krushkrok No29 (2013)krushkrok No29a (2013)

Median nerve arises from the brachial plexus & receives contributions from both the medial & lateral cords. It gives NO branch in the arm area (i.e from origin to cubital fossa). It supplies both pronators (pronator teres & quadratus), and all flexors except the flexor carpi ulnaris & the ulnar half of the flexor digitorum profundus. In the palmar area, the cutaneous branches supply the skin of the palmar surface of fingers 1-3 including the radial aspect of skin of the palmar surface of fingers 1-3 including the radial aspect of the 4th finger.

30. A 38-year-old female patient complains of general weakness, cardiac pain, increased appetite, no menstruation. Objectively: the height is 166 cm, weight 108 kg, the patient has moon-shaped face, subcutaneous fat is deposited mainly in the upper body, torso and hips. There are also blood-red streaks. Ps- 62/min, AP- 160/105 mm Hg. Which of the following diseases is the described pattern of obesity most typical for?

Explanation

\"\\"krushkrok\"\"\\"krushkrok\"

Cushing Syndrome: Etiology:

INCREASE Cortisol due to a variety of causes (Glucocorticoids):

Exogenous corticosteroids: result in decrease ACTH (MCC).

Primary adrenal adenoma, hyperplasia or carcinoma (Cushing’s Syndrome).

ACTH-secreting pituitary adenoma (Cushing Disease).

Findings: Hypertension, Weight Gain, Moon Facies, Truncal Obesity, Buffalo Hump, Skin Changes (thinning striae), Osteoporosis, Hyperglycemia (Insulin resistance), Amenorrhea, Immunosuppression.

31. A 60-year-old patient with a long history of atherosclerosis and a previous myocardial infarction developed an attack of retrosternal pain. 3 days later the patient was hospitalized and then died of progressive cardiovascular insufficiency. At autopsy a white fibrous depressed area about 3 cm in diameter with clear boundaries was found in the posterior wall of the left ventricle and interventricular septum. The dissector evaluated these changes as:

Explanation

Focal cardiosclerosis: Focal –white fibrous depressed area 3cm in diameter. Cardiosclerosis - Atherosclerosis.   

32. Measurements of the arterial pCO2 and pO2 during an attack of bronchial asthma revealed hypercapnia and hypoxemia respectively. What kind of hypoxia occurred in this case?

Explanation

Respiratory hypoxia arises as a result of respiratory insufficiency due to alveolar hypoventilation, disturbances of lung blood supply, disturbances of gases diffusion in lungs. Alveolar hypoventilation may be due to obstructive (bronchial asthma) and restrictive violations of lungs ventilation, which can lead to hypercapnia (INCREASE in CO2) & hypoxemia (DECREASE in O2 in blood).

Tissue or histotoxic hypoxia is the type of hypoxia produced by the inability of tissues to utilize O2 which results from tissue poisoning. These poisonous substances destroy the cellular oxidative enzymes and there is a complete paralysis of cytochrome oxidase system.

 

Hemic hypoxia: connected with hemoglobin (Hb) quantity or inhibition of its functions. Cardiovascular or circulatory hypoxia arises during the disturbance of blood circulation due to heart and vessels pathology. Hypoxic hypoxia develops during the decrease of barometric pressure that is accompanied with decrease of pO2

33. A female patient with bronchial asthma had taken prednisolone tablets (1 tablet 3 times a day) for 2 months. Due to a significant improvement of her condition the patient suddenly stopped taking it. What complication is likely to develop in this case?

Explanation

One of the major setbacks related to continuous, long term steroid (cortisol, prednisolone) treatments are symptoms due to changes in the balance of normal hormone secretion. This typically results from taking doses greater than our body’s natural production. Once we begin to decrease or discontinue the dose, however, withdrawal symptoms may occur. Thus, steroids are typically given for the shortest time possible. Short-term use does not induce steroid withdrawal syndrome. In general, the longer you are on this steroid, the longer it is going to take your body to readjust to functioning without it. If you are on it for longer than 2 weeks, it can affect your adrenal glands ability to produce cortisol.

34. A patient with suspected dysentery has been admitted to the infectious diseases hospital. Which basic method of laboratory diagnosis must be applied in the first place?

Explanation

The laboratory diagnosis of infectious diseases involves two main approaches: one is the bacteriologic approach in which the organism is identified by staining and culturing the organism and the other is the immunologic approach in which the organism is identified by detection of antibodies against the organism in the patient’s serum. When cultures are negative (i.e bacteriologic method), then immunologic testing is commonly used.
35. During a surgery with the use of hygronium the patient had an abrupt fall in blood pressure. Blood pressure can be normalized by the representatives of the following drug group:

Explanation

Hygronium is a ganglioblocker. They can cause dilatation of arteries and veins that leads to an DECREASE of arterial pressure. To normalize blood pressure (BP), we can use alpha-adrenergic agonists which will cause constriction of smooth muscles in peripheral arterioles. INCREASE in peripheral resistance will lead to INCREASE BP.

Alpha-blockers and Ganglio-blockers will further lead to hypotension. M&N-cholinergic agonist does not have significant effect on BP.

36. A patient with respiratory failure has blood pH of 7,35. pCO2 test revealed hypercapnia. Urine pH test revealed an increase in the urine acidity. What form of acid-base imbalance is the case?

Explanation

Respiratory acidosis is caused by alveolar hypoventilation. During hypoventilation, the lungs fail to expel CO2 (hypercapnia). pH of 7.35 indicates a compensatory mechanism that keeps the pH within the normal range. (7.34-7.45 is the normal range) COMPENSATED will remain in the normal range. DECOMPENSATED will fall out of the normal range.

Metabolic acidosis is characterized by excess accumulation of organic acids (lactic acid, ketoacids, uric acid) in the body. Respiratory alkalosis is caused by alveolar hyperventilation which causes excess loss of CO2 from the body.

37. On examination a patient was found to have medial strabismus, the inward deviation of the eyeball and inability to abduct the eyeball outwards. What nerve is damaged?

Explanation

krushkrok No37 (2013)

Lateral rectus muscle: Abducts the eyeball (pulls it away from the midline) & it’s supplied by the abducent nerve (CNVI) which abducts the eye, pulling it laterally. A lesion of CNVI will lead to failure of eye to abduct.

38. A patient with a dislocated shoulder had been admitted to a hospital. With the purpose of skeletal muscle relaxation he was given an injection of relaxant dithylinum acting normally 5-7 minutes. However, the effect of dithylinum in this patient lasted up to 8 hours. What is the most likely cause of the prolonged effect of dithylinum in this patient?

Explanation

Dithyllium is a depolarizing myorelaxant (neuromuscular blocking drug). They are used as adjuvant drugs in anaesthesia during surgery to relax skeletal muscles. An absence or deficiency of acetylcholinesterase/pseudocholinesterase/butyrlcholinesterase enzyme leads to a silent condition that manifests itself only when people that have the deficiency receive the muscle relaxants. This enzyme plays an important role in the metabolism of ester-based local anesthetics, a deficiency of blood cholinesterase can prolong the effect of dithylinium from just 5-7 minutes to 8 hours.

39. As a result of an injury of the knee joint a patient shows a drawer sign, that is the anterior and posterior displacement of the tibia relative to the femur. What ligaments are damaged?

Explanation

krushkrok No39 (2013)

The cruciate ligaments are located in the middle of the joint . The ligaments serve to stabilize the femur and tibia. We have the anterior and posterior cruciate ligament. Drawer test is performed by pulling the tibia anteriorly and posteriorly with a flexed knee. Injury to the cruciate ligament will show a drawer sign with abnormal laxity of the tibia.

40. The neurosurgical department has admitted a 54-year-old male complaining of no sensitivity in the lower eyelid skin, lateral surface of nose, upper lip. On examination the physician revealed the inflammation of the second branch of the trigeminal nerve. This branch comes out of the skull through the following foramen:

Explanation

krushkrok No40 (2013)krushkrok No91 (2014)

Infraorbital nerve supplies inferior eyelid, external lateral surface of nose, upper lip and the cheek. It is the largest branch of Maxillary nerve(CN V2) – 2nd branch of trigeminal nerve. Maxillary nerve escapes from the cranial cavity via the foramen rotundum (round foramen) – which leads to the Pterygopalatine fossa.

41. Bacteriological examination of purulent discharges from the urethra revealed some gram-negative bean- shaped bacteria located in the leukocytes. They can be identified as the causative agent of the following disease:

Explanation

Gonorrhea is caused by Neisseria gonorrhoeae. Its often intracellular (within neutrophils), gram-negative diplococci. It is sexually or perinatally transmitted.

Syphillis is caused by Treponema pallidum(spirochetes); Chancroid is caused by Haemophilus ducreyi; Trichonomoniasis is caused by trichomonas vaginalis; Veneral lymphogrnaulomatosis is caused by Chlamydia trachomatis (L1-L3). They are all sexually transmitted.

Specific identification of the gonococcus can be made either by its fermentation of glucose (but not maltose) or by fluorescent-antibody staining.

42. A male patient is 28 years old. Histological study of a cervical lymph node revealed a change of its pattern due to the proliferation of epithelioid, lymphoid cells and macrophages having nuclei in form of a horseshoe. In the center of some cell clusters there were non-structured light- pink areas with fragments of nuclei. What disease are these changes typical for?

Explanation

FullSizeRender (7)When tubercle bacilli are introduced into the tissue, they are ingested by the alveolar macrophage. Macrophages undergo changes resembling epithelial cells – EPITHELOID cells. Some of the macrophages form MULTINUCLEATED GIANT cells by fusion of adjacent cells (langerhan’s or foreign body type). The giant cells may have 20 or more nuclei. These nuclei may be arranged at the periphery like HORSE-SHOE, RING or clustered at the poles or they may be present centrally (foreign body giant cells). Lymphocytes, plasma cells and fibroblasts surround the epitheloid cells and giant cells (hard tubercle- no central necrosis). Within 10-14 days, the centre of the cellular mass begins to undergo caseation necrosis – soft tubercle. This is the hallmark of tuberculous lesions.
43. A 35-year-old male patient has been referred by an andrologist for the genetic counselling for the deviations of physical and mental development. Objectively: the patient is tall, has asthenic constitution, gynecomastia, mental retardation. Microscopy of the oral mucosa cells revealed sex chromatin (single Barr body) in 30% of cells. What is the most likely diagnosis?

Explanation

\"krushkrokBarr body is an inactive X-chromosome. So a boy (XY) with an inactive X-chromosome must have an additional X-chromosome – XXY (Klinefelter’s syndrome). Causes :

* nondisjunction (maternal and paternal nondisjunction in meiosis I)

* Mosaicism: with the karyotype being 46, XY/47, XXY

Manifestations: gynecomastia, female pattern of pubic hair distribution, no facial hair, high voice. KlinefelterXXY (one barr body) \"FullSizeRender
44. A patient with jaundice has high total bilirubin that is mainly indirect (unconjugated), high concentration of stercobilin in the stool and urine. The level of direct (conjugated) bilirubin in the blood plasma is normal. What kind of jaundice can you think of?

Explanation

Hemolytic/Prehepatic Jaundice is the type of Jaundice that occurs because of excessive destruction of RBCs resulting in ↑ blood level of free, Indirect, unconjugated bilirubin. In this condition, the excretory function of the liver is normal. But the quantity of bilirubin ↑ enormously. The liver cells cannot excrete that much excess bilirubin rapidly. Formation of Urobilinogen (stercobilin) ↑ resulting in the excretion of more amount of stercobilin in stool and urine.

Indirect; Hemolytic; Prehepatic

Mixed; Parenchymal; Hepatic

Direct; Obstructive; Mechanic; Posthepatic

Stercobilin (faeces)

↑↑↑

Decreases (pale faces)

Absent (clay coloured faeces)

Type of bilirubin in blood

Unconjugated

Conjugated and Unconjugated

Conjugated

  • Gilbert syndrome (familial nonhemolytic Jaundice): Autosomal recessive defect. Impaired UGT activity. Jaundice occurs with fasting, volume depletion, stress, menses.

  • Physiologic Jaundice of newborn: begins on day 3 of life. Caused by normal macrophage destruction of fetal RBCs containing HbF and inability of the newborn’s liver to handle excess load.

45. A male with a lesion of one of the CNS parts has asthenia, muscular dystonia, balance disorder. Which CNS part has been affected?

Explanation

During cerebellar lesions, there  are disturbances in posture, equilibrium and movements. Disturbances in movements: Speech disorders, ataxia, asynergia, asthenia (weak muscle contractions with characteristically rapid onset of muscle fatigue), Dysmetria, Intention tremor, astasia (loss of the capacity for sustained tetanic contractions), nystagmus, rebound phenomenon, dysarthria, adiadochokinesis, atonia (lack or impairment of muscle tone).

NB: Scanned or staccato speech is as a result of cerebellar disorders; while monotonus speech is as a result of parkinson's disease.

46. A 50-year-old patient has been administered laevomycetin for the treatment of typhoid fever, but on the next day the patient’s condition worsened, the temperature rose to 39, 60C . The deterioration of the patient’s condition can be explained by:

Explanation

Endotoxins: outer cell membrane of most gram negative bacteria. It is not secreted from the cell. It is the structural part of bacteria (lipopolysaccharide) released when the bacteria is lysed. It was originally theorized that endotoxin is released once the bacteria dies. It is now a known fact that bacteria release small amounts of endotoxin as a part of their normal metabolism although the majority is still retained inside the cell. So when the laevomycetin is used, it kills the bacteria but the deterioration of the patient’s condition is as a result of the endotoxin released when the bacteria died. Effects of endotoxin: fever, hypotension, edema, DIC. 

47. A 12-year-old patient has been admitted to a hospital for hemarthrosis of the knee joint. From early childhood he suffers from frequent bleedings. Diagnose the boy’s disease:

Explanation

Hemophilia is a genetic disorder. Its an intrinsic coagulation pathway defect. Since its genetic, it can manifest early in life (childhood). Types:

Hemophilia A: X-linked recessive; deficiency of factor VIII

Hemophilia B: X-linked recessive; deficiency of factor IX

Hemophilia C: Autosomal recessive; deficiency of factor XI

Findings: macrohemorrhage in hemophilia – hemarthroses (bleeding into joints, such as knee), easy bruising, bleeding after trauma or surgery (e.g. dental procedures).

Idiopathic or Immune thrombocytopenic purpura: characterized by bleeding which results from unusually low levels of platelets, affects both and children and adults. Children often develop Idiopathic thrombocytopenic purpura after a viral infection and usually recover fully without treatment. In adults, however the disorder is often chronic.

 

Hemorrhagic vasculitis is an inflammatory disorder characterized by a generalized vasculitis involving the small vessels of the skin, GI tract, kidneys, joints and rarely the lungs and CNS. It is the most common vasculitis in children. It is an immune complex HSR disease; inciting agents (antigens) include: group A β-hemolytic streptococci and other bacteria, viruses, drugs, food, insect bites.

48. Examination of a patient with ischemic heart disease revealed the impaired venous blood flow in the territory of the cardiac vein running in the anterior interventricular sulcus of heart. What vein is it?

Explanation

coronary_veins1coronary_veins2.jpg-w=490

* Vena magna cordis (Great cardiac vein): originates in the area of the apex of the heart. The vein runs along the anterior interventricular sulcus, turns left and enters the coronary sulcus.

* Vena cordis media (middle cardiac vein): also originates on the apex of the heart. It ascends along the posterior interventricular sulcus and joins the coronary sinus next to its opening.

* Vena cordis parva (small cardiac veins): situated within the right portion of coronary sinus. It originates from the area of right ventricle and runs leftwards to reach the coronary sinus.

* Vena(e) posterior ventriculi sinistri [posterior vein(s) of left ventricle]: originates from several small veins on the posterior surface of left ventricle and flows either into the coronary sinus or into the terminal portion of great cardiac vein.

 

* Vena oblique atria sinistri (oblique vein of left atrium): runs obliquely along the posterior surface of left ventricle and flows into the coronary sinus next to the great cardiac vein.

These 5 veins are the veins related to the coronary sinus.

49. For the direct injection of medications into the liver surgeons use the round ligament of liver. This manipulation involves bougienage (lumen dilatation) of the following vessel:

Explanation

Round ligament of liver (ligamentum teres hepatis): it runs from the visceral surface to the navel; it is an obliterated peritoneum–enfolded umbilical vein. The orifice of the vein may remain partially patent (open) and thus the vein can be used for infusion of drugs and radiopaque agents. The umbilical vessels closure occurs during the first week of life; the umbilical vein transforms into peritoneum–enfolded round ligament of liver. The umbilical arteries also close and transform into the medial umbilical ligaments. The single umbilical vein is responsible for carrying oxygenated blood from placenta to fetus in fetal circulation.

50. A patient with lobar pneumonia has had body temperature of 39oC with daily temperature fluctuation of no more than 1oC for 9 days. This fever can be characterized by the following temperature curve:

Explanation

krushkrok No50 (2014)

Febris continua/persistent: elevated temperature persists at a high level, difference between morning and evening temperature does not exceed 10C.

* Febris intermittens: characterized by regular alternation of brief attacks of fever (paroxysms) with feverless periods (apyrexia). Attacks occur every 3rd day, 2nd day or everyday. Increase temperature persists for several hours, drops to normal and then rises again.

* Febris recurrens: characterized by longer periods of pyrexia than intermittent (5-6days). Question says every 4 days.

* Febris hectic: 3-50C (difference in temperature).

* Febris remittens: difference in temperature exceeds 10C but temperature never falls to normal.

51. The temperature in a production room is 36oC . Relative air humidity is 80%. Under these conditions the human body transfers heat mainly through:

Explanation

Evaporation is a way the body dissipates heat to the environment by its evaporation via sweat or evaporation of moisture from the skin and respiratory tract mucous membranes of (“wet” heat loss). Evaporation closely related to relative humidity.

Heat Radiation is a way the surface of the human body emits heat to the environment in the form of infrared rays. The amount of heat the body radiates to the environment is proportional to the surface of radiation area and to the difference between the mean values of skin and environment temperature. The surface radiation area is the total surface area of body parts that contact the air. Elimination of heat by radiation increases with a decrease in ambient temperature and decreases with its increase. It is possible to reduce elimination of heat by radiation via reduction of the surface of radiation area (“winding oneself into a ball”). Heat radiation does not require a medium for transfer of heat. (Key words: naked or lightly clothed).

Convection is a way the body eliminates heat by means of transferring heat via moving particles of air or water. To dissipate heat by means of convection, body surface shall be airflowed at a temperature that is lower than the temperature of the skin. At that, air layer contacting with the skin warms up, decreases its density, rises and is replaced by cooler, denser air. By increasing the speed of the air flow (wind, ventilation) heat emission increases significantly as well (forced convection). Convection requires convection current; current of gases or liquids (Key words: air over exposed area of skin).

 

Conduction is a way the body eliminates heat by means of direct contact with another object. Heat is transferred down the temperature gradient (i.e. from the object of higher temperature to the object of lower temperature). Conduction requires contact with another object (Key words:  in water). 

52. A hospitalized patient bitten by a rabid animal has an avulsive wound of shin. What kind of vaccine must be given to prevent rabies?

Explanation

As the name implies, anti-rabies vaccine is used to prevent rabies. Rabies is a viral disease of animals and humans can be infected by animal’s bite (most common – Dog). A dog can be immunized against the disease.

 

BCG – Tuberculosis; DTaP – vaccine against Diphtheria, Tetanus and Pertussis (whooping cough); Td – vaccine against Tetanus and Diphtheria; TABte – vaccine against enteric fever (abdominal typhus, paratyphoid) and Tetanus.

53. At autopsy the occipital lobe of brain was found to have a cavity 2,5x1,5 cm large filled with a transparent liquid. The cavity had smooth brownish walls. What process had developed in the brain?

Explanation

Cyst cavity contains transparent fluid, blood, tissue or tumor cells; Abscess cavity contains pus. Both with a distinguishable wall.

54. A child entering the school for the first time was given Mantoux test in order to determine if there was a need for revaccination. The reaction was negative. What is the meaning of this test result?

Explanation

Tuberculin (Mantoux) skin test: this test is done by intradermal injection of tuberculoprotein (tuberculin), purified protein derivative (PPD). Type IV hypersensitivity reaction.

Immunization against tuberculosis is induced by injection of attenuated strains of bovine type of tubercle bacilli, Bacilli Calmette Guerin (BCG). 

The Mantoux skin test should be read between 48 and 72hrs after administration. The basis of reading  is the presence or absence of induration, which may be determined by inspection and by palpation. A record should also be made of formation of vesicles, bullae, lymphangitis, ulceration and necrosis at the test site. The formation of vesicles, bullae or necrosis at the test site indicates positive result. A negative mantoux result usually signifies that the individual has never been exposed to Mycobacterium tuberculosis i.e. absence of cell mediated immunity to tuberculin.

Mantoux test is a type IV Hypersensitivity reaction (HSR), which involves macrophages,T-lymphocytes and lymphokines(cytokines). Mononuclear cells (lymphocytes,monocytes,macrophages).

55. Study of the biopsy material revealed a granuloma consisting of lymphocytes, plasma cells, macrophages with foamy cytoplasm (Mikulicz cells), many hyaline globules. What disease can you think of?

Explanation

Microscopic examination of specific granulomas:

* In Rhinoscleroma of nose, the granuloma (scleroma) consists of plasma cells, epitheloid cells, lymphocytes and hyaline sphere. Large macrophages with light cytoplasm containing klebsiella rhinoscleromatis (Mikulicz’s cells), sclerosis and hyalinosis takes place.

* In TB, the granuloma is reffered to as a tubercle and is classically characterized by the presence of central necrosis surrounded by epitheloid cells, lymphocytes, plasma cells and giant langhance cells. In contrast, caseous necrosis is rare in other granulomatous diseases.

* The syphilis granuloma is calle Gumma. Gumma consist of a central area of fibrinoid or caseous necrosis surrounded by mononuclear inflammatory cells, mostly plasma cells, lymphocytes, epitheloid cells and seldom-giant langhance cells. Around gumma forms the granulation tissue and endovasculitis.

* In Tuberculoid Leprosy, the epidermis contains confluent granulomas composed of macrophages, plasma cells and leprous Virchow’s cells – Leprous Virchow’s cells (or Leprous cells) refer to large foamy macrophages within fatty vacuoles containing leprous mycobacterium.

 

* Actinomycosis caused by Actinomyces. It occurs rarely in human but rather frequently in cattle. Characterized by the formation of painful abscess. Infected man often have poor oral hygiene or recent dental work. Does not form granulomas.

56. Autopsy of a 78-year-old patient revealed that retroperitoneal tissue was soaked with blood, the abdominal aorta had a sacciform protrusion including a defect with irregular edges. The wall of the aorta was here and there of stone-like density. This is the complication of the following disease:

Explanation

Atherosclerosis of aorta is the most common form of atherosclerosis in which the aorta aneurysm usually develop. Most common aneurysm associated with atherosclerosis is the saccular aneurysm (sacciform protrusion). Wall of aorta – stone-like density as a result of scarring from Atherosclerosis.

 

Essential hypertension affects basically the small muscular arteries and arterioles.

57. Glycogen polysaccharide is synthesized from the active form of glucose. The immediate donor of glucose residues during the glycogenesis is:

Explanation

The following steps occur in glycogen synthesis: glucose activation; initiation of glycogen synthesis; chain elongation; introduction of branch points. Glucose activation consists of the formation of UDP-glucose from glucose-6-phosphate, which is converted to glucose 1-phosphate by the enzyme phosphoglucomutase. Glucose 1-phosphate is then activated to UDP-glucose by glucose 1-phosphate uridylyltransferase; this reaction uses uridine triphosphate (UTP). UDP-glucose is the substrate in both the initiation step and the repetitive chain elongation steps. The enzyme responsible for the initiation and extension of the linear polymer is glycogen synthase.

58. After the diagnostic tests a 40-year-old male has been referred for the lymphography of the thoracic cavity. The surgeon revealed that the tumor had affected an organ whose lymphatic vessels drain directly into the thoracic duct. Specify this organ:

Explanation

The thoracic part of the thoracic duct resides anterior to the vertebral column, in between the aorta and the azygos vein and posterior to the esophagus. Lymphatic vessels of the esophagus may pass directly to the thoracic duct or to the posterior mediastinal nodes. Lymphatic vessels of the trachea drain into pretracheal, paratracheal and inferior deep cervical nodes to the jugular trunk → thoracic duct. Pericardium is drained by parasternal nodes, lateral pericardial nodes and prepericardial nodes. Heart is drained by brachiocephalic nodes, nodes of arch of azygos, node of ligamentum arteriosum. The bronchopulmonary nodes and tracheobronchial nodes drain the left main bronchus. The syntopy of the esophagus allow direct drainage into the thoracic duct (both lie on the left part of thoracic cavity in the posterior mediastinum).

59. A patient with biliary dyskinesia and constipations has been prescribed a cholagogue having also a laxative effect. What drug has been administered?

Explanation

Allochol and cholenzyme are agents of biliary acids and bile (cholesecretics – induce bile secretion). Cholosas is a plant agent (cholesecretics – induce bile secretion). Nicodinum is a synthetic agent (cholesecretics – induce bile secretion). They all stimulate bile formation. Magnesium sulfate is a drug causing gall bladder contraction and relaxation of sphincter of oddi; it promotes bile excretion. It is also a salt laxative acting on all intestine’s secretions. It is a non-absorbable salt that hold water in the intestine by osmosis and distend the bowel, increasing intestinal activity and producing defecation.

60. It is known that individuals with genetically caused deficiency of glucose 6-phosphate dehydrogenase may develop RBC hemolysis in response to the administration of some antimalarial drugs. Manifestation of adverse reactions to drugs is called:

Explanation

Idiosyncrasy and allergic reaction is as a result of organism’s increased sensitivity to the drugs. It’s a complication of pharmacological therapy (adverse reactions). Tolerance and Tachyphylaxis are effects of repeated doses.

* Idiosyncrasy is genetically conditioned perverse organism’s response to drugs (e.g. inefficiency, lack of enzymes participating in drug metabolism). Genetic deficiency of glucose 6-phosphate dehydrogenase (hereditary enzymopathy). This enzyme is needed for proper functioning of glutathione (an antioxidant) to prevent the oxidative effect of antimalarial drugs on the RBCs.

* Allergic reactions results from antibody production against some drugs.

* Tachyphylaxis is quick weakening of the effect, sometimes may occur after first drug introduction (usually connected with substrate exhaustion).

 

* Tolerance (habituation): is increasing of the effects in repeated doses resulting from decreasing of absorption, speed-up of the biotransformation or excretion, or decreasing of the receptors sensitivity.

61. A 40-year-old patient with the progressing staphylococcal purulent periodontitis developed purulent inflammation of bone marrow spaces of the alveolar process, and then of the body of mandible. Microscopy revealed thinning of bone trabeculae, foci of necrosis, bone sequesters surrounded by the connective tissue capsule. What is the most likely diagnosis?

Explanation

Chronic osteomyelitis: most common causative agent is staphylococcus aureus. Characterized by bone necrosis (sequestrum formation), low-grade inflammation, persistent and progressing infection.  Also characterized by low-grade clinical presentations.

62. Curariform substances introduced into a human body cause the relaxation of all skeletal muscles. What changes in the neuromuscular synapse cause this phenomenon?

Explanation

The first drug that was found to be capable of blocking the skeletal neuromuscular junction (NMJ) was curare, which native South American hunters of the Amazon region used to paralyze prey. The drug tubocurarine was ultimately purified and introduced into clinical practice in the early 1940s. Muscarinic and Nicotinic receptors are the two cholinergic receptors (cholinoreceptors). Nicotinic receptors are located in the CNS, adrenal medulla, autonomic ganglia and the NMJ. Those at the NMJ are the ones that are blocked by Curariform substances.

63. A number of diseases can be diagnosed by evaluating activity of blood transaminases. What vitamin is one of cofactors of these enzymes?

Explanation

Vitamin B6 (pyridoxine): coenzyme forms –

*Pyridoxal phosphate (PALP)

*Pyridoxamine Phosphate (PAMP)

 

Components of pyridoxal enzymes used in transamination (e.g. ALT and AST); decarboxylation reactions, glycogen phosphorylase, transformation of tryptophan to Niacin (Vit B3), active transport of amino acids through the cell membrane, synthesis of cystathionine, heme, histamine and neurotransmitters including serotonin, epinephrine, norepinephrine, dopamine and GABA.

64. After a car accident a 23-year-old male presented to the hospital with a cut wound of the anteromedial region of shoulder and arterial bleeding. Which artery was damaged?

Explanation

The subclavian artery becomes the axillary artery at the lateral border of the first rib. The axillary artery becomes the brachial artery at the inferior borders of Pectoralis major (anteriorly) and Latissimus dorsi (posteriorly). Therefore, a cut wound of the anteromedial region of shoulder will damage the brachial artery. Subscapularis is the greatest branch of axillary artery. Radial artery and ulnar artery are terminal branches of brachial artery in the cubital fossa. Profunda is the greatest branch of brachial artery arising from its upper portion.

65. During the operation on the small intestine the surgeon revealed an area of the mucous membrane with a single longitudinal fold among the circular folds. Which portion of the small intestine is this structure typical for?

Explanation

image

The small intestine: duodenum, jejunum, ileum

Duodenum: superior part, descending part, inferior part and ascending part. The descending part of the duodenum runs inferiorly, curving around the head of pancreas. The bile and main pancreatic ducts enter its posteromedial wall. These ducts usually unite to form the hepatopancreatic ampulla, which opens on an eminence called the major duodenal papilla (papilla duodeni major) located posteromedially in the descending duodenum.

66. 14 days after quinsy a 15-year-old child presented with morning facial swelling, high blood pressure, \"meat slops\"urine. Immunohistological study of a renal biopsy sample revealed deposition of immune complexes on the basement membranes of the capillaries and in the glomerular mesangium. What disease developed in the patient?

Explanation

Peritonsillar abscess also known as quinsy ( a complication of tonsillitis) caused by both aerobic and anaerobic bacteria (streptococcus, staphylococcus and haemophilus).

Acute poststreptococcal glomerulonephritis: most frequently seen in children. Occurs approximately 2 weeks after group A streptococcal infection of pharynx or skin. Resolves spontaneously. Type III hypersensitivity reaction (Immune complex). Presents with peripheral and periorbital edema, cola-coloured urine, hypertension. On immunofluorescent microscopy: granular appearance due to IgG, IgM and C3 deposition along glomerular basement membrane and mesangium. On electron microscopy: subepithelial immune complex humps. On light microscopy: glomeruli enlarged and hypercellular. 
67. A diseased child has a high fever, sore throat, swelling of submandibular lymph nodes. Objectively: pharyngeal mucosa is edematous, moderately hyperemic, the tonsils are enlarged, covered with grayish membrane tightly adhering to the tissues above. Attempts to remove the membrane produce the bleeding defects. What di- sease are these presentations typical for?

Explanation

Diphtheria is an acute infectious disease characterized by fibrinous inflammation in the focus of primary fixation of the causative agent and general intoxication due to exotoxin absorption. Causative agent – Corynebacterium diphtheria. Local changes in Diphtheria in the pharynx: gray or white patches of exudates appear on the pharyngeal mucosa, usually over the tonsils. These enlarge and coalesce and with the accumulation of blood, become gray or black. The lymphoid tissues both in regional lymph nodes and systematically (as in the spleen) undergo hyperplasia with the development of prominent germinal centers that are often centrally necrotic. The epithelial surface becomes necrotic and easily adherent to the overlying membrane; this adherence explains why raw bleeding points are exposed when the membrane is forcibly removed. The soft tissue of the neck is swollen. In severe toxic forms, the edema is considerable and can involve the anterior surface of the chest.

There are two types of fibrinous inflammation: croupous and diphtheric fibrinous inflammation. Usually croupous inflammation develops on the columnar epithelium. In this case the fibrinous membranes unfix easily, without any effort. Diphtheric fibrinous inflammation develops on the squamous or intermediate epithelium and the fibrinous membranes unfix with difficulties and may even bleed when trying to unfix it.

68. Study of the biopsy material of an embryo revealed a zone of developmental abnormality in a somite. The zone was located close to the endoderm and the notochord. What formations may have abnormal development in case of pregnancy continuation?

Explanation

The embryonic period or period of organogenesis occurs from the third – eighth weeks of development and is the time when each of the 3 germ layers: ectoderm, mesoderm and endoderm, give rise to a number of specific tissues and organs.

Mesoderm: paraxial, intermediate and lateral mesoderm. By the beginning of the 3rd week, paraxial mesoderm begins to be organized into segments. These segments, known as somitomeres, which further organize into somites (42-44 pairs). Somites: sclerotome (tendon, cartilage and bone – vertebrae and rib cage); Myotome (muscles); Dermatome (dermis). From their initial location within the somite, the sclerotome cells migrate medially towards the notochord.
69. A smear of sputum from the patient with suspected lobar pneumonia was stained with the use of the following stains and reagents: solution of gentian violet, Lugol’s solution, 96o alcohol, water magenta. What staining method was applied in this case?

Explanation

Gram stain is used to differentiate bacterial species into two large groups (gram positive and gram negative) based on the physical properties of their cell walls. Gram positive bacteria retain the crystal violet dye (gentian violet), while the counter stain (red/magenta) added after the crystal violet gives all gram negative bacteria a red or pink colour. It requires 4 basic steps and 4 staining agents are added sequentially: Gentian/Crystal violet; Lugol’s solution (Iodine); 96% alcohol (ethanol); water magenta (counter stain).

Ziehl-Neelsen (carbol fuchsin): Acid fast bacteria (Mycobacteria – Tuberculosis); protozoa.

Romanovsky Giemsa: Trypanosomes, Plasmodium, Chlamydia, Borrelia, Rickettsia

 

Neisser stain: filamentous bacteria

70. A patient has normally coloured stool including a large amount of free fatty acids. The reason for this is a disturbance of the following process:

Explanation

A normally coloured stool indicates that there is no obstruction to bile flow, therefore, there is a normal emulsification of fat. This also indicates that there is no obstruction to pancreatic juice flow into the intestine, therefore, there will also be proper digestion (hydrolysis) of fat by pancreatic lipase. Then, the only way free fatty acid can be found in stool after emulsification and digestion is only when there is a problem with absorption by the intestinal villi.

71. Examination of the removed stomach revealed a deep roundish defect with regular edges at the lesser curvature of the antrum. The defect reached the muscular tunic and was 1,5 cm in diameter. Within the defect floor there was a translucent dense area resembling of a hyaline cartilage. What process had developed in the floor of the stomach defect?

Explanation

Hyalinosis (hyaline changes): transparent, glass-like, usually refers to an alteration within cells or in the extracellular matrix which gives a homogenous, glassy, pink appearance. It can be classified according to its localization (vascular and connective tissue hyalinosis) and propagation (generalized and localized). It is localized because it has a diameter of 1.5cm; it did not affect a large portion or the whole stomach wall.

72. By the decarboxylation of glutamate in the CNS an inhibitory mediator is formed. Name it:

Explanation

image
73. Thermometry revealed that the temperature of the exposed skin is by 1-1,5oC lower than the temperature of the adjacent areas covered with clothing from natural fabrics. The reason for this is that the clothes reduce the heat loss through:

Explanation

Convection is a way the body eliminates heat by means of transferring heat via moving particles of air or water. To dissipate heat by means of convection, body surface shall be airflowed at a temperature that is lower than the temperature of the skin. At that, air layer contacting with the skin warms up, decreases its density, rises and is replaced by cooler, denser air. By increasing the speed of the air flow (wind, ventilation) heat emission increases significantly as well (forced convection). Convection requires convection current; current of gases or liquids (Key words: air over exposed area of skin). 

The areas of the skin covered with clothes has a higher temperature because the convection current does not have direct access to the skin in that area as compared to the exposed area of the skin.

Heat Radiation is a way the surface of the human body emits heat to the environment in the form of infrared rays. The amount of heat the body radiates to the environment is proportional to the surface of radiation area and to the difference between the mean values of skin and environment temperature. The surface radiation area is the total surface area of body parts that contact the air. Elimination of heat by radiation increases with a decrease in ambient temperature and decreases with its increase. It is possible to reduce elimination of heat by radiation via reduction of the surface of radiation area (“winding oneself into a ball”). Heat radiation does not require a medium for transfer of heat. (Key words: naked or lightly clothed).

Evaporation is a way the body dissipates heat to the environment by its evaporation via sweat or evaporation of moisture from the skin and respiratory tract mucous membranes of (“wet” heat loss). Evaporation closely related to relative humidity.

Conduction is a way the body eliminates heat by means of direct contact with another object. Heat is transferred down the temperature gradient (i.e. from the object of higher temperature to the object of lower temperature). Conduction requires contact with another object (Key words:  in water). 

74. A specimen of pia mater includes a vessel whose wall doesn’t have the tunica media, the tunica externa is adherent to the surrounding tissues, the intima is composed of a basement membrane and endothelium. What vessel is it?

Explanation

The tunics of veins are not as distinct or well defined as the tunics of arteries. Veins are divided into 3 types:

* Small veins/venules: postcapillary and muscular venules

* Medium veins

* Large veins

Arteries:

·        Large or elastic arteries

·        Medium or muscular arteries

·        Small arteries and arterioles

Muscular venules are distinguished from postcapillary venules by the presence of a tunica media. Postcapillary venules possess an endothelial lining with its basal lamina and pericytes. Postcapillary venules have no true tunica media.

·        Tunica intima: consists of endothelium with its basal lamina

·        Tunica media: smooth muscle cells

·        Tunica adventitia: collagen fibers

A vessel without tunica media, also lack muscular tissue. Muscular artery, arteriole and artery of mixed type all have tunica media. Only capillaries and postcapillary venules lack tunica media.

 

Also a prominent internal elastic membrane helps to distinguish muscular arteries from elastic arteries and muscular venules. 

75. A patient with extensive burns of torso skin exhibits signs of severe intoxication. What stage of the burn disease is this typical for?

Explanation

Intoxication – burn toxemia

The complex of typical changes in the human body developing due to the deep and severe burns is known as burn disease. There are the following stages of burn disease: burn shock, burn toxemia, burn infection, burn emaciation, outcome.

Burn toxemia is related to accumulation in the organism of toxic products of tissue decomposition and systemic expression of multiple inflammatory mediators (systemic inflammatory response syndrome).

76. As a result of a craniocerebral injury a patient has a decreased skin sensitivity. What area of the cerebral cortex may be damaged?

Explanation

IMG_9921

krushkrok No46a (2014)

Postcentral gyrus (i.e. part of parietal lobe immediately posterior to central sulcus) is concerned with sensory information (touch).

Precentral gyrus (i.e. part of frontal lobe immediately anterior to central sulcus) is concerned with motor information. Occipital lobe: visual cortex. Cerebellum: coordination of movement and postural adjustment. Temporal lobe: superior temporal gyrus – located in temporal lobe immediately below sylvian fissure concerned with auditory stimuli. Hippocampus - memory

77. A histological specimen of the eyeball shows a biconvex structure connected to the ciliary body by the fibers of the Zinn’s zonule and covered with a transparent capsule. Name this structure:

Explanation

* Lens: a transparent, crystalline, biconcave structure; suspended from the inner surface of the ciliary body by a ring of radially oriented fibers, the zonule of zinn. Diopter is a unit of measurement of the optical power of a lens or curved mirror.

* Vitreous body: composed of a transparent gel substance that fills the vitreous chamber. It contains hyaluronic acid, widely dispersed collagen fibrils and other proteins and glycoproteins. The fluid component of the vitreous body is called vitreous humor.

* Cornea: anterior window of the eye.

* The cornea is continuous with the sclera. The sclera is composed of dense fibrous connective tissue that provdes attachment for the extrinsic muscles of the eye. It constitutes the “white” of the eye.

 

* The ciliary body is a ring-like thickening that extends inward just posterior to the level of the corneoscleral junction. Within the ciliary body is the ciliary muscle, a smooth muscle that is responsible for lens accommodation. Contraction of the ciliary muscle changes the shape of the lens, which enables it to bring light rays from different distances to focus on the retina.

78. A comatose patient was taken to the hospital. He has a history of diabetes mellitus. Objectively: Kussmaul breathing, low blood pressure, acetone odor of breath. After the emergency treatment the patient’s condition improved. What drug had been administered to the patient?

Explanation

The described symptoms are for type I diabetes mellitus (especially acetone odor of breath as a result of excess ketone bodies). Type I diabetes mellitus is due to decrease or absent insulin. Insulin facilitates the transport of glucose and amino acids into target organs: In liver – insulin increases the storage of glucose as glycogen. It decreases protein catabolism; In muscles – it stimulates glycogen synthesis and protein synthesis; In Adipose tissue – it facilitates triglyceride storage by activating plasma lipoprotein lipase which increase glucose transport into the cell and by reducing intracellular lipolysis.

Glibenclamide is typically used for type 2 diabetes mellitus.

 

Adrenaline will further exacerbate the hyperglycemic condition because it stimulates glycogen phosphorylase. Isadrinum is a non-selective β12 adrenergic agonists (β-adrenomimetics); it’ll have the same effect as adrenaline. Furosemide is a loop diuretic and will not correct the comatose state of the patient; It has no effect on glucose metabolism or ketone bodies.

79. In order to stimulate breathing in a child born with asphyxia, the doctor gave him a drug injection into the umbilical vein. What drug might have been injected?

Explanation

Analeptics (aethimizolum): drugs which act upon the CNS general functions, but stimulate mainly medulla oblongata (respiratory, cardiovascular centers). Aethimizolum acts on the medulla with a direct action. 

Aethimizolum also activates the adrenocorticotropic pituitary function which increase cortisol level → ↑production of surfactant. All other options are analeptics but they are only used in the absence of aethimizolum in case of children born with asphyxia.

80. A patient complains of pain in the right lateral abdomen. Palpation revealed a dense, immobile, tumor-like formation. A tumor is likely to be found in the following part of the digestive tube:

Explanation

imageimage

Anterolateral abdominal wall has 9 regions and 4 quadrants (RUQ, LUQ, RLQ, LLQ)

·        Right lateral abdominal region: Ascending colon (colon ascendens), right kidney, right ureter and loops of small intestine.

·        Umbilical region: Transverse colon (colon transversum), head of pancreas, duodenum (except superior part)

·        Left lateral abdominal region: Descending colon (colon descendens), left kidney, left ureter and loops of small intestine.

·        Left inguinal region: Sigmoid colon (colon sidmoideum), left ureter, left external iliac artery of artery and vein.

·        Right Inguinal region: Caecum, vermiform appendix, right ureter

81. A patient underwent biopsy of the soft palate arches for a suspected tumor (macroscopy revealed an ulcer with a dense floor). Study of the biopsy material revealed mucosal necrosis with infiltration of lymphocytes, epithelioid cells, plasma cells, single neutrophils in the submucosa. There were also apparent signs of endovasculitis and perivasculitis. The described changes are typical for:

Explanation

Primary syphilis: the chancre develops at the site of inoculation in 10-90days (average 21 days) and has a characteristic “luetic vasculitis” (endovasculitis and perivasculitis). The chancre is a slightly elevated, firm, reddened papule that erodes to create a clean-based, shallow ulcer. Histologically, the chancre contains an intense infiltrate of plasma cells, with scattered macrophages and lymphocytes and an obliterative endarteritis. The regional lymph nodes are usually enlarged and may show nonspecific acute or chronic lymphadenitis, plasma cell-rich infiltrates or focal epitheloid granulomas. The combination of chancre, lymphangitis and lymphadenitis is called – primary syphilitic complex.

82. Healthy parents with unremarkable family history have the child with multiple developmental defects. Cytogenetic analysis revealed the trisomy 13 in the somatic cells (Patau syndrome). What phenomenon has caused the defects?

Explanation

In preparation for fertilization, germ cells undergo gametogenesis, which includes meiosis, to reduce the number of chromosomes. Chromosomal abnormalities may be numerical or structural. Abnormalities in chromosome number may originate during meiotic or mitotic divisions. In meiosis, two members of a pair of homologous chromosomes normally separate during the first meiotic division, so that each daughter cell receives one member of each pair. Sometimes, however, separation does not occur (nondisjunction) and both members of a pair move into one cell. As a result of nondisjunction of the chromosomes, one cell receives 24 chromosomes and the other receives 22 instead of the normal 23. When at fertilization a gamete having 23 chromosomes fuses with a gamete having 24 or 22 chromosomes, the result is an individual with either 47 chromosomes (e.g.trisomy 13, 18,21, klinefelter) or 45 chromosomes (monosomy – 45 XO turner’s syndrome). Nondisjunction which occurs during either the first or the second meiotic division of the germ cells, may involve the autosomes (trisomy 13,18,21) or sex chromosomes (turner’s and klinefelter’s syndrome). 

 

If nondisjunction occurs during mitosis (mitotic nondisjunction) – such conditions produce mosaicism, with some cells having an abnormal chromosome number and others being normal. Therefore abnormal gametogenesis can produce nondisjunction.

83. A specimen shows an organ covered with the connective tissue capsule with trabeculae radiating inward the organ. There is also cortex containing some lymph nodules, and medullary cords made of lymphoid cells. What organ is under study?

Explanation

Lymph nodes are small encapsulated organs located along the pathway of lymphatic vessels. The supporting elements of the lymph node are:

 *Capsule: composed of dense connective tissue that surrounds the node.

*Trabeculae: composed of dense connective tissue, which extend from the capsule into the substance of the node, forming a gross framework.

*Reticular tissue: composed of reticular cells and reticular fibers that forms a fine supporting meshwork throughout the remainder of the organ.

The parenchyma of the lymph node is divided into a cortex and medulla. The cortex consists of lymphocytes which are organized into nodules. The medulla of the lymph node consists of the medullary cords and medullary sinuses.

Tonsils: form a ring of lymphatic tissue at the entrance of the oropharynx; consisting of aggregations of lymphatic nodules.

Thymus is a lymphoepithelial organ located in the superior mediastinum. It possess a thin connective tissue capsule from which trabeculae extend into the parenchyma of the organ. The trabeculae establish domains in the thymus called thymic lobules. Thymic or hassall’s corpuscles are a distinguishing feature of the thymic medulla.

Spleen is the largest lymphatic organ. Most of the spleen consists of splenic pulp. Splenic pulp is divided into white pulp and red pulp. The spleen is enclosed by a dense connective tissue capsule from which trabeculae extend into the parenchyma of the organ.

Red bone marrow lies entirely within the spaces of bone in the medullary cavity of young long bones and the spaces of spongy bone.
84.

A   25-year-old   patient  consulted a doctor  about  dysmenorrhea and infertility. Examination revealed  that  the patient was 145 cm high and had underdeveloped secondary sex characteristics, alar folds on the  neck.  Cytological  study  didn’t reveal any Barr bodies in the somatic cells. What diagnosis was made?

Explanation

FullSizeRender (1)

krushkrok No105 (2014)

These characterize Turner’s syndrome (XO). It can be complete monosomy (45,XO) or mosaicism (e.g. 45,XO/46,XX). Klinefelter is related to males. The rest are not related to sex chromosomes, but they are autosomoes.

Barr body is an inactive X-chromosome. A normal female has one barr body XX; so in turner’s syndrome, there is no barr body because the only X present is the active one (45 XO).

85. To prevent attacks  of acute  pancreatitis a doctor  prescribed the patient trasylol (contrycal, gordox),  which is an inhibitor of:

Explanation

Acute pancreatitis is an autodigestion of pancreas by pancreatic enzymes (proteolytic enzymes). Contrycal (Aprotinin, Gordox)- protease inhibitor. Therefore,it can inhibit the proteolytic enzymes digesting the pancreas.  Contrycal also has antifibrinolytic (antiplasmin) and hemostatic effect. 
86.

A patient died from progressive heart failure.  Autopsy revealed  that  the  heart was enlarged in diameter, flabby. The muscle  section  exhibited  irregular blood supply. Histological  study of myocardium revealed hyperemia, the stroma was found to  have  lymphohistiocytic  infiltrates  with  degeneration of  cardiomyocytes.  The revealed  morphological changes  are indicative of:

Explanation

Hyperemia and lymphohistiocytic infiltrates indicates inflammation. Its not myocardial infarction nor cardiosclerosis because no necrosis is involved. No fatty infiltration, therefore its not cardiomyoliposis. Therefore, the best answer is Non-purulent interstitial myocarditis.
87.

A 13-year-old  teenager underwent X-ray examination of the  hip joint.  Examination  revealed  a 3mm wide radiolucent zone  between the  head  and  the  shaft  of femur.  This situation should  be evaluated as:

Explanation

Normally all of femoral ossification centres fuse between the ages 14-18years old. Therefore a radiolucent zone between the head and shaft of femur at age 13 years old is normal.

88.

A surgeon  examined the  patient and found  the injury of the upper  third of the kidney.  Considering  the  syntopy  of  the left kidney, the intactness of the following organ should be checked at the same time:

Explanation

The left kidney neighbors the left suprarenal gland, the pancreas, the stomach, the left colic flexure and the small intestine. The stomach is the best answer in the options given when reffering to the upper third of the kidney; the small intestine lies much lower in the abdominal cavity.

89.

A patient with urolithiasis has unbearable  spasmodic   pain.  To  prevent pain shock, he has been given an injection of atropine along with a narcotic analgesic having  antispasmodic  effect.  What  drug was it?

Explanation

All options listed are all opoid analgesics; but Promedol remains the best answer because it has a spasmolytic effect (antispasmodic). Therefore, it is very effective for pains associated with spasms of smooth muscles of internal organs and blood vessels. In comparison with morphine hydrochloride, it has less oppressing activity on respiratory centers, less than excites the center of the vagus nerve and the vomiting center. When the pain is associated with spasms of smooth muscle (angina pectoris, liver, kidney, intestinal cramps) you can assign promedol with atropine, metacin, papaverine etc.

90.

Despite the  administration of  cardiotonics   and  a  thiazide   diuretic   a  patient  with  chronic  heart  failure  has  persistent  edemata, there  is a risk  of ascites. What  medication should  be administered in order  to enhance the diuretic  effect of the drugs used?

Explanation

IMG_9896

The answer is Spironolactone because after the administration of cardiotonics and a thiazide diuretic, if the edema still persists, it is probably due to an increased action of aldosterone on the principal cells in collecting tubule of the kidney. In this case, an aldosterone antagonist will enhance the diuretic effect to prevent the occurrence of ascites. Blocking aldosterone effect prevent further reabsorption of Na+ and H2O which will definitely enhance the diuretic effect. Aldosterone acts on mineralocorticoid receptor → mRNA → protein synthesis (synthesis of Na+ channels). Spironolactone prevents/blocks the synthesis of Na+ channels.

91.

A   patient   with   a   pathology   of the cardiovascular system developed edemata of the  lower  extremities. What is the mechanism  of cardiac edema development?

Explanation

The main symptoms of right-sided heart failure are fluid accumulation and swelling (edema) of the feet, ankles, legs, liver and abdomen. Where the fluid accumulates depends on the amount of excess fluid and the effects of gravity. If a person is standing, fluid accumulates in the legs and feet; if a person is lying down, fluid usually accumulates in the lower back; if the amount of fluid is large, fluid also accumulates in the abdomen.

 

Most water leakage occurs in capillaries or postcapillary venules which have a semipermeable membrane wall that allows water to pass freely than proteins. In cardiac insufficiency, blood pools in the veins since the heart is not pumping effectively; this increases the hydrostatic pressure in the veins. And this increased hydrostatic pressure leads to increase filtration of water into the interstitium at the venous end of the capillary (post-capillary venule).

92. During  the  fight, a man  had  a cardiac arrest  due  to  the  strong  blow  to  the upper  region  of  the  anterior abdominal  wall. Which of the following mechanisms has led to the cardiac arrest?

Explanation

Goltz’ reflex: by pressing/strong blow to the epigastric region produces parasympathetic responses. Aschner’s reflex: press on the eyeball produces similar parasympathetic effects. They are parasympathetic reflexes and because no prior learning is involved – it is an unconditioned reflex.

93. A  pregnant woman  underwent AB0 blood typing. Red blood cells were agglutinated with  standard sera  of the  I and II blood groups, and were not agglutinated  with the  III  group  serum.  What  is the patient’s blood group?

Explanation

Blood group O(I): no antigens, therefore no agglutination.

Blood group A(II): A antigen, agglutinate with blood group B(III) and O(I).

Blood group B(III): B antigen, agglutinate with blood group A(II) and O(I).

Blood group AB(IV): A and B antigen, agglutinate with all blood groups. No antibody.

Blood group name is determined by the antigen present on RBC, but the patient has an opposite antibody. So whenever, the antibody corresponds to the antigen, there is agglutination. Since O does not have any antigen, no agglutination can occur.

94.

Amniocentesis   revealed    two    sex chromatin bodies  (Barr  bodies)  in  each cell  of  the  sample.  What  disease  is this character typical for?

Explanation

Barr body is an inactive X-chromosome. A normal female has one barr body XX, a normal male has no barr body XY.

Trisomy XXXX (only one X is active in a female; therefore, 2 barr bodies)

KlinefelterXXY (one barr body)

Turner’s – XO (no barr body)

 

Down and Patau involves autosomal chromosomes and not sex chromosomes.

95.

A hospital  has admitted a patient complaining  of abdominal bloating, diarrhea,  flatulence   after   eating   protein foods. These signs are indicative of the impaired digestion  of proteins and  their increased  degradation. Which of the following compounds is the product of this process?

Explanation

Normally, protein digestion takes place in the stomach and small intestine. When this process is impaired, the  protein goes to the large intestine undigested and the gut microflora degrades/breaks down protein into ammonia, phenols, indoles and amines which have been shown to exert toxic effects in vitro and in animal models. Human studies have shown that colonic protein metabolism via the gut microflora is responsive to dietary protein as faecal ammonia and urinary phenolic compound concentrations increase in response to increased intake of protein rich foods. These gases also cause the abdominal bloating, diarrhea and flatulence mentioned in the question.

96.

An attack of tachycardia that occurred in a patient was stopped by pressing on his eyeballs.  Which  of the  following  reflexes underlies this phenomenon?

Explanation

Aschner’s reflex (press on eyeball) → ↓heart rate. This is mediated by nerve connections between the ophthalmic branch of trigeminal cranial nerve via the ciliary ganglion and the vagus nerve of parasympathetic nervous system.

Goltz reflex (press or blow to the epigastric region) → ↓heart rate.

Goltz’ reflex: by pressing/strong blow to the epigastric region produces parasympathetic responses. Aschner’s reflex: press on the eyeball produces similar parasympathetic effects. They are parasympathetic reflexes and because no prior learning is involved – it is an unconditioned reflex.

97. A    male    patient   has    been    diagnosed with acute post-streptococcal glomerulonephritis. It  is most  likely that the   lesion  of  the   basement  membrane  of  renal   corpuscles   was  caused   by  the following allergic reaction:

Explanation

FullSizeRender (17) Acute poststreptococcal glomerulonephritis: most frequently seen in children. Occurs approximately 2 weeks after group A streptococcal infection of pharynx or skin. Resolves spontaneously. Type III hypersensitivity reaction (Immune complex). Presents with peripheral and periorbital edema, cola-coloured urine, hypertension. On immunofluorescent microscopy: granular appearance due to IgG, IgM and C3 deposition along glomerular basement membrane and mesangium. On electron microscopy: subepithelial immune complex humps. On light microscopy: glomeruli enlarged and hypercellular.
98.

An  unconscious  patient was taken  by ambulance  to  the  hospital.   On  objective  examination  the   patient  was  found to  have  no  reflexes,  periodical   convulsions, irregular breathing. After  laboratory examination the patient was diagnosed with   hepatic    coma.   Disorders  of   the central nervous system develop due to the accumulation of the following metabolite:

Explanation

Substances absorbed into the bloodstream from the intestine pass through the liver, where toxins are normally removed. Many of these toxins (such as ammonia) are normal breakdown products of the digestion of protein. In hepatic encephalopathy (hepatic coma), toxins are not removed because liver function is impaired. Ammonia is produced by amino acid metabolism and intestinal urease-positive bacteria. In physiological conditions, it is mostly present as ammonium (NH4+) in serum. The urea or ornithine cycle, which is fully expressed in the liver exclusively, serves to converts NH4+ to urea prior to renal excretion and to maintain low serum concentrations. In hepatic coma, when the liver cannot remove toxins and urea cycle is not functional, all this occurs:

NH3 + α-ketoglutarate → Glutamate

α-ketoglutarate is used up which leads to:

·        ↑glutamate → ↑GABA (inhibitory neurotransmitter)

·    Inhibition of citric acid cycle/tricarboxylic acid cycle; this causes impairment of ATP formation.

·        Inhibition of metabolism of amino acids (impairment of transamination reactions).

 

NH3 + Glutamate → Glutamine

Glutamine is an amide of glutamic acid which provides a non-toxic storage and transport form of ammonia (NH3). Ammonia increase synthesis of glutamine in brain. Accumulation of glutamine in brain results in elevation of osmotic pressure in nervous cells leading to brain edema.

                                   

                                    NH3 + H+ → NH4+

In blood ammonia (NH3) is represented as ammonium ion (NH4+). Accumulation of ammonium ion impairs transport of ions (Na+, K+) through cell membranes and failure of transmission of nerve impulse.

Urea cycle takes place exclusively in the liver, so in hepatic coma, urea level is low. Glutamine toxicity in brain is dependent on increased ammonia concentration.

 

Bilirubin toxicity will most likely be related to increase hemolysis, which is not the case in this question. Histamine is a biogenic amine produced from the amino acid histidine.

99.

A  20-year-old  male  patient complains  of  general   weakness,   rapid   fatigability, irritability, decreased performance, bleeding   gums,  petechiae  on  the   skin. What  vitamin  deficiency  may be a cause of these changes?

Explanation

Vitamin C (ascorbic acid): found in fruits and vegetables; an antioxidant; also facilitates iron absorption by reducing it to Fe2+ state. It is necessary for hydroxylation of proline and lysine in collagen synthesis; necessary for dopamine β-hydroxylase, which converts dopamine to norepinephrine. Deficiency leads to: scurvy – swollen gums, bruising, petechiae, hemarthrosis, anemia, poor wound healing, perifollicular and subperiosteal hemorrhages, “corkscrew” hair; Weakened immune response.

Type III collagen is found in blood vessels; Type IV collagen is found in basement membrane. Deficiency in Vitamin C disrupts the second stage of collagen synthesis in fibroblasts (hydroxylation of collagen) which results in petechiae, bruising, hemarthrosis.

Vitamin B2 (riboflavin) deficiency – growth retardation, glossitis, conjunctivitis

Vitamin B1 (thiamine) deficiency – Beri-Beri (polyneuritis)

Vitamin A (retinol) deficiency – Night blindness

 

Vitamin B9 (folic acid) deficiency – macrocytic megaloblastic anemia

100.

It   is  known   that   the   monoamine oxidase (MAO) enzyme plays an important part in the metabolism of catecholamine neurotransmitters. In what way does the enzyme inactivate  these neurotransmitters (norepinephrine, epinephrine, dopamine)?

Explanation

image

image

Norepinephrine (noradrenaline), epinephrine (adrenaline) and dopamine are catecholamines produced in chromaffin cells of adrenal medulla from tyrosine. The catecholamines are inactivated by oxidative deamination catalyzed by monoamine oxidase (MAO) and by O-methylation carried out by Catechol-O-methyltransferase. The metabolic products of these reactions are excreted in the urine as vanillymandelic acid from epinephrine and norepinephrine; and homovanillic acid from dopamine.

101. The  cellular  composition of exudate largely  depends on the  etiological  factor of inflammation. What leukocytes  are the first to get into the focus of inflammation caused by pyogenic bacteria?

Explanation

Neutrophils are the first leukocytes that cross the blood vessel wall to enter inflammatory sites. Under normal conditions, leukocytes are restricted to the center of small blood vessels, where the flow is fastest. In inflammatory sites, where the vessels are dilated, the slower blood flow allows the leukocytes to move out of the center of the blood vessel and interact with the vascular endothelium. Even in the absence of infection, monocytes migrate continuously into the tissues, where they differentiate into macrophages; meanwhile, during an inflammatory response, the induction of adhesion molecules on the endothelial cells, as well as induced changes in the adhesion molecules expressed on leukocytes recruit large numbers of circulating leukocytes, initially Neutrophils and later monocytes, into the site of an infection (inflammatory focus).

 

First –Neutrophils; second –monocytes and macrophages; third –lymphocytes.

102.

At  the  end  of  the  working  day  a worker  of a hot work shop has been delivered to a hospital. The patient complains of a headache, dizziness, nausea,  general weakness. Objectively: the patient is conscious, his skin is hyperemic, dry, hot to the touch. Heart rate is of 130/min. Respiration  is rapid,  superficial.  What  disorder of thermoregulation is most likely to have occurred in this patient?

Explanation

The patient was in a hot workshop, there is definitely a transfer of heat from the environment to the patient. Compensatory mechanisms in the patient should work to increase heat transfer from the patient back to the environment (heat emission), to avoid accumulating excessive heat. The patient was hyperemic (vasodilation –normal), dry (i.e. not sweating) and hot to touch (if there was evaporation of sweat, it would have cooled the skin temperature). In the absence of sweating, it therefore means that there is reduced heat transfer from the patient’s body back into the surrounding.

 

NB: the question is concerned about heat transfer from the patient to the surrounding.

103.

Alveolar   space    of    the    acinus was invaded by some bacteria which interacted with  the  surfactant.  This  led to  the   activation   of  the   cells  that   are localized  in the alveolar  walls and on the surface. What cells are these?

Explanation

Alveolar macrophages remove inhaled particulate matter from the air spaces and red blood cells (RBCs) from the septum of alveoli. They are unusual in that they function both in the connective tissue of the septum (alveolar wall) and in the air space of the alveolus (surface of alveolar cells). In air spaces they scavenge the surface to remove inhaled particulate matter (e.g. dust, pollen, pathogens), thus giving them one of their alternate names – Dust cells. They also phagocytose infectious organisms such as Mycobacterium tuberculosis. Other macrophages remain in the septal connective tissue, where, filled with accumulated phagocytized material, they may remain for much of an individual’s life. Thus, at autopsy, the lungs of urban dwellers as well as smokers will usually show many alveolar and septal macrophages filled with carbon particles, anthracotic pigment and birefringent needle-like particles of silica.

Type I alveolar cells (type I pneumocytes): the junctions formed between this cells form an effective barrier between the air space and the components of the septal wall. They are not capable of cell division. They make up 95% of the surface of the alveoli.

Type II alveolar cells: secretory cells; have lamellar bodies; progenitor cells for type I alveolar cells.

Clara cells: non-ciliated; low columnar/cuboidal cell with secretory granules. They secrete components of surfactant, degrade toxins and act as reserve cells.

 

Endothelial cells line blood vessels.

104.

A 35-year-old  male developed acute heart  failure  while running  for a long time. What  changes  in the ionic composition can be observed  in the cardiac muscle?

Explanation

Increase in cardiac workload can cause imbalances in oxygen (O2) supply and demand to the heart itself. Blood supply to the heart occurs during diastole. Increase heart rate (↑HR) → ↓diastolic period → ischemia in myocardium. Ischemia, hypoxia, heart failure are associated with disruptions in intracellular Na+ and Ca+ concentration homeostasis of myocardial cells. A decrease efflux or increase influx of Na+ may cause cellular Na+ overload. Na+ overload is followed by an ↑influx of calcium (Ca2+) through Na+-Ca2+ exchanger. Failure to maintain the homeostasis of Na+ and Ca+ leads to electrical instability (arrhythmias), mechanical dysfunction (reduced contractility and ↑ diastolic tension) and mitochondrial dysfunction. These events increase ATP hydrolysis and decrease ATP formation and if left uncorrected, they cause cell injury and death → acute heart failure.

105. Workers of a conveyor  workshop received recommendations for the effective organization of working  time  and  higher  working  efficiency.  What  peculiarity of work in this workshop  causes the greatest stress for the workers?

Explanation

Stress is not a disease or injury but it can lead to mental and physical ill health. We are concerned about the mental health in this question. Now the workers have been recognized and appreciated for a job well done which is a fundamental human need. The stress related hazard in this work now will be the job content (monotony), in which they are made to do the same thing always and repeatedly. This leads to unhappiness and boredom.

106.

Mother of  a boy  who  had  recently returned  from   a  summer   camp   found some  small  whitish  insects  up  to  3 mm long  on  the  child’s clothing.  Specify  the parasite:

Explanation

Pediculus humanus humanus (body or clothes louse). The insect was found on the clothing.

107.

Histological  examination of the removed skin neoplasm  revealed  clusters and cords of atypical cells of stratified squamous epithelium, growing into the underlying tissue. What  diagnosis  can be assumed?

Explanation

The epidermis of skin is composed of stratified squamous epithelium. It is nonkeratinizing because it is growing into the underlying tissue. It probably would have being keratinized if it is growing out of the tissue.

108.

An 18-year-old male has been diagnosed with Marfan syndrome. Examination    revealed    a   developmental   disorder  of connective tissue  and  eye  lens structure, abnormalities of the cardiovascular  system,  arachnodactylia. What genetic phenomenon has caused the development of this disease?

Explanation

* Pleiotropy: one gene contributes to multiple phenotypic effects → disorder of connective tissue and eye lens structure, abnormalities of CNS, arachnodactylia. Marfan is FBN1 gene mutation on chromosome 15; Marfan is one gene defect causing all this phenotypic defects.

* Codominance: both alleles contribute to the phenotype of the heterozygote. e.g. blood group AB

* Incomplete dominance: intermediate inheritance. Red + White → Pink

 

* Multiple allelism e.g. ABO blood type

109.

A patient has severe catarrhal symptoms.  Material growth on Bordet-Gengou   agar    showed    mercury-drop-like  colonies.  Examination of  the  blood smears  revealed  some  small ovoid  gram-positive bacilli sized 1-3 microns. What microorganisms were isolated?

Explanation

Bordet-Gengou agar and Regan-Lowe medium: media for isolation of Bordetella pertussis.

Tellurite agar/Lὄffler medium – corynebacterium diphtheria

Lὄwenstein-Jensen agar – M.tuberculosis; Eaton agar – M. pneumonia

Thayer-Martin agar – Neisseria meningitides/gonorrhoeae

 

Brucella agar – Brucella

110. A 66-year-old patient with Parkinson’s disease  shows an improvement in locomotor  activity   after   prolonged  use of  a  certain  drug  which  is converted to dopamine by the  decarboxylation. What drug has the patient taken?

Explanation

Levodopa is a metabolic precursor of dopamine. Because parkinsonism results from insufficient dopamine in specific regions of the brain, attempts have been made to replenish the dopamine deficiency. Dopamine itself does not cross the blood-brain barrier, but its immediate precursor – levodopa, is actively transported into the CNS and is converted to dopamine in the brain. Large doses of levodopa are required, because much of the drug is decarboxylated to dopamine in the periphery, resulting in side effects that include nausea, vomiting, cardiac arrhythmias and hypotension.

Celecoxib: NSAIDs (selective COX-2 inhibitor); indications – rheumatoid arthritis

Droperidol: typical neuroleptic; indications – psychoses, schizophrenia

Chlorpromazine (aminazine): same group and indication as droperidol

 

Naloxone : opoid antagonist; indications – acute poisoning with narcotic analgesics.

111.

Enzymatic  jaundices  are accompanied  by  abnormal activity  of  UDP-glucuronyl   transferase. What  compound is accumulated in blood  serum  in case of these pathologies?

Explanation

In the hepatocyte, the solubility of unconjugated bilirubin is increased (i.e. it is made soluble) by the addition of two molecules of glucuronic acid to produce conjugated bilirubin. This process is reffered to as conjugation. This reaction is catalyzed by Uridine diphosphate (UDP) glucuronyltransferase – UGT. Varying degrees of deficiency of this enzyme result in Crigler-Najjar I & II and Gilbert syndrome; with Crigler-Najjar I being the most severe deficiency.

Deficiency of UGT inhibits conjugation and therefore increase unconjugated bilirubin in serum (Jaundice).

112.

For   the   study   of  serum   proteins various physical and physicochemical methods  can be used. In particular, serum albumins  and  globulins  can be separated by this method:

Explanation

Electrophoresis uses the principle of electric charge size and shape to separate substances. Proteins carry a positive or a negative electrical charge and they move in fluid when placed in an electrical field. The two major types of protein present in the serum are albumin and globulin proteins. Albumin is the major protein component of serum and represents the largest peak that lies closest to the positive electrode. Globulins comprise a much smaller fraction of the total serum protein but represent the primary focus of interpretation of serum protein electrophoresis. Five (5) globulin categories are expressed – α1, α2, (alpha 1 & 2), β1, β2 (beta 1 & 2) and ɣ (gamma). ɣ is the closest to the positive electrode.

113. Negative  environmental factors have caused   the   dysfunction   of  myosatellite cells. What  function  of the  whole muscle fibre is likely to be changed  in this case?

Explanation

Satellite cells are interposed between the plasma membrane of the muscle fiber and its external lamina. They are small cells with scant cytoplasm. Satellite cells function as stem cells that, after injury, proliferate to give rise to new myoblasts. As long as the external lamina remains intact, the myoblasts fuse within the external lamina to form myotubes, which then mature into a new fiber.

114.

The  laboratory  for  especially dangerous infections  conducts  microscopic examination of pathological material from  a  patient  with  suspected plague. The sample was stained  by Burri-Gins technique. What property of the causative  agent  can be  identified  by this technique?

Explanation

Burri-Gins technique – capsule formation; Ozheshko – spores

115. Autopsy of a 62-year-old woman revealed  a dense well-circumscribed node of 6 cm in diameter in the  cranial  cavity. The node  was attached to the dura  mater and histologically consisted of clusters and micro-concentric structures of endothelial cells, psammoma bodies.  What  kind of tumor  was found at autopsy?

Explanation

Psammoma bodies are laminated concentric spherules with dystrophic calcification. PSaMMoma bodies are seen in:

·       * Papillary carcinoma of thyroid

·        *Serous papillary cystadenocarcinoma of ovary

·        *Meningioma

·        *Malignant mesothelioma

 

Meningioma is the only one in the options given.

116.

Inherited diseases, such as mucopolysaccharidoses,   are   manifested in  metabolic disorders   of  connective tissue, bone  and  joint  pathologies. The  sign of this disease  is the excessive urinary excretion of the following substance:

Explanation

The Mucopolysaccharidoses are hereditary disorders that are clinically progressive. They are characterized by accumulation of glycosaminoglycans in various tissues, causing varied symptoms such as skeletal and extracellular matrix deformities and mental retardation. Mucopolysaccharidoses are caused by a deficiency of any one of the lysosomal hydrolases normally involved in the degradation of heparin sulfate and/or dermatan sulfate. This results in the presence of oligosaccharides in the urine because of incomplete lysosomal degradation of glycosaminoglycans.

117. An  animal  has  an  increased   tonus of extensor muscles.  This is the  result  of enhanced information transmission to the motoneurons of the  spinal  cord  through the following descending pathways:

Explanation

Vestibular nuclei receive impulses concerned with muscle tone and posture from vestibular apparatus and cerebellum. Vestibular nuclei in turn convey the impulses to different parts of the body through the anterior and lateral vestibulospinal tracts. Vestibulospinal tracts are concerned with adjustment of position of head and body during angular and linear acceleration; maintenance of muscle tone and posture; position of head and body during acceleration. Therefore, increase transmission of impulse through this tract leads to increase tone. Extensor muscles are also for balance (connected to vestibular apparatus). The inputs from the otolith organs project mainly to the lateral vestibular nucleus, which in turn sends axons in the lateral vestibulospinal tract to the spinal cord. The input from this tract exerts a powerful excitatory influence on the extensor (antigravity) muscles. When hair cells in the otolith organ are activated, signals reach the medial part of the ventral (anterior, motor) horn. By activating the ipsilateral (same side) pool of motor neurons innervating extensor muscles in the trunk and limbs, this pathway mediates balance and the maintenance of upright posture.

118.

A   specimen    of   a   parenchymal organ shows poorly  delineated hexagonal lobules  surrounding a  central  vein,  and the  interlobular connective tissue contains embedded triads (an artery, a vein and an excretory duct). What organ is it?

Explanation

krushkrok No118 (2013)

krushkrok No118a (2013)

There are 3 ways to describe the structure of the liver in terms of a functional unit: the classic lobule; portal lobule and the liver acinus. The classic lobule is the traditional way to describe the organization of the liver parenchyma. The classic hepatic lobule is a roughly hexagonal (6-sided) mass of tissue. At the center of the lobule is a relatively large venule, the terminal hepatic venule (central vein), into which the sinusoids drain. At the angles of the hexagon are the portal areas (portal canals), loose stromal connective tissue characterized by the presence of the portal triad. The portal triad is composed of the branches of the hepatic artery, portal vein and the bile duct.

119.

A   patient  has   been   admitted  to the infectious  diseases department for malaise,  fever  up  to  38oC ,  jaundice.   A few  months  ago,  the  patient underwent blood  transfusion. The  doctor  suspected viral hepatitis B. What  are  the  principal methods of laboratory diagnosis of hepatitis B?

Explanation

Hepatitis B virus (HBV): incubation period 30-180days; it’s a DNA virus. Transmission: sexual, blood, vertical transmission via pregnancy and breast feeding. Primarily spread by blood, accidental needle sticks. HBV diagnosis is accomplished by testing for a series of serological markers of HBV. Nucleic acid testing for HBV-DNA is increasingly being used to quantify HBV viral load and measure the effectiveness of therapeutic agents.

120.

After   resection of  the  middle  third of the  femoral  artery  obliterated by a thrombus the limb is supplied  with blood through the  bypasses.  What  artery  plays the  main  part  in  the  restoration of  the blood flow?

Explanation

krushkrok No120 (2013)

The femoral artery arises directly from the external iliac artery. The boundary between the arteries is the inferior border of the inguinal ligament. The greatest branch of the femoral artery is the deep artery of thigh. The deep femoral artery (deep artery of thigh) is nearly as wide as the femoral artery. It arises 4-5cm below the inguinal ligament. It is the main artery which supplies the thigh. Its branches are medial circumflex, lateral circumflex femoral arteries; and perforating arteries. Clinical applications: the deep artery of thigh is of great importance for collateral circulation. Intrinsic anastomoses and intersystem anastomoses (with the internal iliac and popliteal arteries) provide good conditions for collateral circulation, which is vital in occlusions of femoral artery.

121.

During   the   intravenous  transfusion  of  the   saline   the   patient’s   condition  deteriorated dramatically, and  the patient died  from  asphyxiation. Autopsy revealed  acute  venous  congestion of internal  organs   with   the   dramatic  right heart  dilatation. When the right ventricle     was    punctured   underwater, the bubbles escaped. What pathological process occurred in the patient?

Explanation

Embolism is occlusion of a vessel by material travelling in the circulation.

* Gas embolism: two main forms of gas embolism are air embolism and decompression sickness. Air embolism is usually due to accidental pumping of air into the venous circulation during intravenous (IV) injection or transfusion ( bubble – air escaped).

* Fat/Adipose embolism: obstruction of arterioles and capillaries by fat globules constitutes fat embolism. It may occur following severe fracture trauma to bones, inflammation of bones and soft tissues, fatty liver

* Thromboembolism: a detached thrombus or part of thrombus which may arise in the arterial or venous circulation.

 

Tissue embolism: fragments of tissue.

122.

At the post-mortem examination the stomach of a patient with renal failure was found  to have a yellow-brown  coating  on the  thickened mucosa.  The  coating  was firmly adhering to its surface  and  had  significant thickness. Microscopy revealed congestion  and  necrosis  of mucosal  and submucosal  layers,  fibrin presence. What is the most likely diagnosis?

Explanation

Fibrinous inflammation is an inflammatory response of mucous surface (oral, respiratory, bowel) to toxins of diphtheria or irritant gases. As a result of denudation of epithelium, plasma exudes on the surface where it coagulates and together with necrotized epithelium, forms false membrane that gives this type of inflammation its name. Histologically, fibrin appears as an eosinophilic network of threads or sometimes as an amorphous coagulum.

123.

Infectious diseases  are  treated  with antibiotics  (streptomycin, erythromycin, chloramphenicol). They inhibit the following stage of protein synthesis:

Explanation

Streptomycin (aminoglycoside) is a 30S ribosomal subunit inhibitor.

Erythromycin (macrolide); Chloramphenicol – 50S subunit inhibitor

 

Protein synthesis inhibitors specifically target smaller bacterial ribosome (70S –made of 30S and 50S subunits), leaving human ribosome (80S) unaffected. All these drugs blocks the translation of information on mRNA to protein.

124. Diseases  of  the  respiratory  system and  circulatory  disorders  impair  the transport  of   oxygen,   thus   leading   to hypoxia.  Under these  conditions the energy metabolism is carried  out by anaerobic glycolysis. As a result, the following substance  is generated and accumulated in blood:

Explanation

Lactate, formed by the action of lactate dehydrogenase (converting pyruvate to lactate) is the final product of anaerobic glycolysis in eukaryotic cells. In organs or cells that are poorly vascularized and/or lack mitochondria, formation of lactate is the major fate of pyruvate as seen in lens, cornea of the eye, kidney medulla, testes, leukocytes and red blood cells.

Aerobic glycolysis progresses to citric acid cycle from pyruvate. The cycle occurs totally in the mitochondria.

125.

A patient has been hospitalized for a suspected  tumor  of the  prostate. During the surgery, it was revealed  that the tumor invaded  the  bladder. Which  part  of  the bladder was affected?

Explanation

krushkrok No125 (2013)

The neck of the urinary bladder (cervix vesicae) is the narrow portion of fundus, which becomes continuous with the urethra. The urethra begins with the internal urethral orifice within the neck of the bladder, passes through the prostate. As shown in the image above, the cervix of the urinary bladder is the closest part to the prostate gland for metastases of any tumor or infectious agent.

126. A casualty with an injury of the temporal region has been  diagnosed  with epidural hematoma. Which of the arteries is most likely to be damaged?

Explanation

The maxillary artery gives 5 branches within the infratemporal fossa: deep auricular, anterior tympanic, inferior alveolar, accessory meningeal and middle meningeal/membranous arteries. The middle meningeal/membranous artery enters the cranial fossa through the foramen spinosum and gives branches to the dura mater of middle cranial fossa. It is the largest dural branch. The middle meningeal artery is most often damaged in fractures of the temporal bone producing epidural hematoma because it lies on the internal skull surface (pterion) which is the thinnest part of the lateral skull; therefore, it is vulnerable to fractures. The pterion is an area where the frontal, temporal, parietal and sphenoid (greater wing) articulate. krushkrok No126 (2013)
127.

A  19-year-old   male  was  found   to have  an  elevated level  of  potassium   in the  secondary urine.  These  changes  might have  been  caused  by the  increase  in the following hormone level:

Explanation

Aldosterone produced in adrenal cortex (zona glomerulosa): causes increased sodium (Na+) reabsorption; increased potassium and hydrogen ions (↑K+, H+) excretion. They increase sodium (↑Na+) channel and Na+ /K+-pump insertion in principal cells of collecting duct; enhances K+ and H+ excretion by way of principal cell K+ channels and α-intercalated cell H+ ATPases of collecting duct. Therefore, increase in aldosterone → ↑ K+ in urine (excretion) and ↓ Na+ in urine (↑ reabsorption); And decreased aldosterone → ↓ K+ excretion (↓K+ in urine) and ↓ Na+  reabsorption (i.e. ↑Na+ in urine); same effects on sweats glands too.

128.

Analysis  of  the  ECG   revealed   the missing of several PQRST  cycles. The remaining waves and  complexes  are  not changed. Specify the type of arrhythmia:

Explanation

Sinoatrial (SA) block is the failure of impulse transmission from SA node to AV node. It is also called sinus block. It is characterized by periodic missing of the heart beat and pulse beat. The ECG shows periodic missing of the heart complex (neither P-wave nor the QRST complex are recorded); the length of diastole doubles.

129.

ECG  of a patient displays an abnormally  long   R   wave   (up   to   0,18 s).  This  is caused  by  a  decrease in  the conduction velocity of the following heart structures:

Explanation

QRS complex on an ECG graph is also called the initial ventricular complex. Q wave is a small negative wave. It is continued as the tall R wave which is a positive wave. R wave is followed by a small negative wave – S wave. QRS complex is due to depolarization of ventricular musculature. Q wave is due to the depolarization of basal portion of interventricular septum. R wave is due to the depolarization of apical portion of interventricular septum and apical portion of ventricular muscle. S wave is due to the depolarization of basal portion of ventricular muscle near the atrioventricular ring. The Q wave may be absent on an ECG. The R wave corresponds to almost complete excitation of both ventricles. It is the highest wave of the ventricular complex.

130.

6 hours after the myocardial  infarction a patient was found  to have  elevated level  of lactate  dehydrogenase in blood. What  isoenzyme  should  be  expected in this case?

Explanation

Lactate dehydrogenase (LDH) is an enzyme in cardiac cells which can serve as a marker for injured myocardial cells as it leaks out of damaged cell membranes into the bloodstream. It catalyzes the conversion of pyruvate to lactate (a reversible reaction). It has five (5) isoenzymes (isoenzymes catalyze same reaction), but only LDH1 is found in the myocardium.

LDH1 – heart, RBCs and brain; LDH2- reticuloendothelial system; LDH3- lungs; LDH4- kidneys, placenta and pancreas; LDH5- liver and striated muscle.

131.

A  46-year-old   female  is  scheduled for  a  maxillofacial   surgery.  It  is  known that  the  patient is prone  to  high  blood coagulation. What  natural anticoagulant can be used to prevent blood clotting?

Explanation

Heparin is a natural anticoagulant produced in mast cells and basophils. It is an injectable, rapidly acting anticoagulant that is often used acutely to interfere with the formation of thrombi. Heparin is used in the prevention of venous thrombosis and the treatment of a variety of thrombotic diseases such as pulmonary embolism and acute myocardial infarction. Heparin binds to antithrombin III, with the subsequent rapid inactivation of coagulation factors. Antithrombin III inhibits serine proteases, including several of the clotting factors, most importantly, thrombin (factor IIa) and Factor Xa (a-active).

 

Protamine sulphate antagonizes the anticoagulant effects of heparin. The positively charged protamine interacts with the negatively charged heparin forming a stable complex without anticoagulant activity.

132.

A 50-year-old  patient with food poisoning is on a drip of 10% glucose solution. It not only provides the body with necessary  energy,  but  also  performs the function of detoxification by the production of a metabolite that participates in the following conjugation reaction:

Explanation

Substances that cause food poisoning are xenobiotics, since they are foreign to the body. The body removes xenobiotics by xenobiotic metabolism, which takes place in the liver. Hepatic enzymes (e.g. cytochrome P450) first activate them- phase I (by oxidation, reduction, hydrolysis and/or hydration of the xenobiotic) and then conjugating the active secondary metabolite (phase II) with glucuronic acid, sulphuric acid or glutathione, followed by excretion in bile or urine. Glucuronidation is the most common and the most important conjugation reaction (glucuronidation – conjugation with glucuronic acid). 

133. To assess the effectiveness  of breathing in patients, the indicator of functional residual  capacity  is used.  It  includes  the following volumes:

Explanation

Functional residual capacity (FRC) is the volume of air remaining in the lungs after normal expiration (after normal tidal expiration). It includes expiratory reserve volume (ERV) and residual volume (RV).  FRC = ERV + RV

134. It is required to evaluate the level of tissue  excitability.  For  this  purpose one should determine:

Explanation

The larger the stimulus, the greater the depolarization or attempt to reach depolarization or threshold potential. The threshold value controls whether or not the incoming stimuli are sufficient enough to generate an action potential. The threshold potential is the critical level to which the membrane potential must be depolarized in order to initiate an action potential. Individual muscle fibers and tissues generally require different depolarization threshold potential for action potential to occur.

135.

Due   to   the   use   of   poor-quality measles  vaccine  for preventive vaccination, a 1-year-old  child developed an autoimmune  renal  injury. The urine  was found to contain macromolecular proteins. What process of urine formation was disturbed?

Explanation

When blood passes through glomerular capillaries, the plasma is filtered. All substances of plasma are filtered except the plasma proteins and plasma cells, due to their large molecular size which is larger than the slit pores present in the endothelium of capillaries. Glomerular capillary membrane, basement membrane and visceral layer of bowman capsule form the filtration membrane through which glomerular filtration occurs. So, if protein and erythrocytes are increased or present in urine, it has to do with increase in renal filter permeability.
136. A patient has been  administered an anti-inflammatory drug that blocks the action   of   cyclooxygenase.   Specify   this anti-inflammatory agent:

Explanation

FullSizeRender (1)

The nonsteroidal anti-inflammatory drugs (NSAIDs) are a group of chemically dissimilar agents that differ in their antipyretic, analgesic and anti-inflammatory activities. They act primarily by inhibiting the cyclooxygenase (COX 1 and 2) enzymes that catalyze the first step in prostanoid biosynthesis. This leads tondecrease prostaglandin synthesis with both beneficial and unwanted effects. Aspirin is one of the most important NSAID. Its most common side effect is its gastrointestinal (GI) effect. Normally, prostacyclin (PGI2) inhibits gastric acid secretion, whereas PGE2 and PGF stimulate synthesis of protective mucus in both the stomach and small intestine. In the presence of aspirin, these prostanoids are not formed, resulting in increased gastric secretion and diminished mucus protection. This may cause epigastric distress, ulceration, hemorrhage and iron-deficiency anemia. Analgin is also an NSAID, but aspirin (acetylsalicylic acid) is far more preferable and more common pharmacologically.

137.

A pneumonia patient has been administered  acetylcysteine  as   a   part of complex therapy. What principle of therapy  was   taken    into   consideration when applying this drug?

Explanation

Acetylcysteine is an expectorant (mucolytic), which ↑bronchial secretion or reduce its viscosity, facilitating its removal by coughing; they loosen cough which becomes less tiring and more productive. This drugs treats the pathogenesis/development of the disease and not its cause (etiology-etiotropic) or symptoms (symptomatic).

138.

A  26-year-old   female  patient with bronchitis  has been administered a broad spectrum  antibiotic as a causal treatment drug. Specify this drug:

Explanation

Doxycycline is a tetracycline, broad spectrum antibiotic. Bacteriostatic; bind to 30s subunit of ribosome. All other options are not antibiotics.

139.

A  65-year-old   male   suddenly   lost the  vision  in  one  eye  due  to  the  retinal  detachment. The  patient underwent enucleation.    Histological     examination of  the  removed eye  retina   and  choroid revealed   clusters   of   atypical   cells  with marked polymorphism of cells and nuclei,  with  a  moderate number   of  mitoses including the pathological ones. The cell  cytoplasm   and   intercellular  medium contained brown pigment giving a positive  DOPA  reaction. Perls’ reaction was negative.  What  is the  most  likely diagnosis?

Explanation

Melanoma is strongly linked to acquired mutations caused by exposure to ultraviolet (UV) radiation in sunlight. Melanomas show striking variations in colour, appearing in shades of black, brown, red, dark blue and gray. The borders of melanomas are irregular and often notched, unlike the smooth, round and uniform borders of melanocytic nevi. The positive DOPA reaction indicates the presence of melanocytes. Melanoma arises in the skin, meninges, uvea of the eye.

140.

A  child  cut  his leg with  a piece  of glass while playing and was brought  to the clinic for the  injection  of tetanus toxoid. In   order   to   prevent  the   development of anaphylactic shock the serum was administered by Bezredka method.  What mechanism  underlies this method of desensitization of the body?

Explanation

Desensitization as a method of treating allergic diseases is used, in particular, to prevent serum anaphylaxis from repeated injections of serum preparations (e.g. tetanus toxoid), in accordance with the method proposed in 1907 by the Russian scientist A. M. bezredka. The method consists in injecting small concentrations of the preparation (toxoid antigen) that produce the sensitization, as a result, a state of anti-anaphylaxis arises (i.e. desensitization). Owing to this, the next injection of the reacting dose of the allergen does not produce anaphylaxis. The small concentrations injected are below threshold concentrations inducing complete and sustained deprivation of releasability (ability to release inflammatory mediators from activated mast cells having IgE attached to them already) without triggering degranulation. Basophil desensitization is regulated by their surface IgE levels.

 

Desensitization therapy in atopic individuals involves repeated injections of increasingly greater amounts of allergens, resulting in production of IgG antibodies that attach to allergens and prevent them from binding to mast cells.

141.

Microscopy   of  the  myocardium  of a  patient who  had  died  from  heart  failure revealed foci of fibrinoid necrosis located  diffusely in the interstitial stroma, and    often    around   the    vessels.   Such foci were surrounded by lymphocytes, macrophages,    histiocytes.     Pericardium was found  to have signs of sero-fibrinous pericarditis. What  is the  most  likely  diagnosis?

Explanation

Rheumatic fever has 4 stages:

· *Mucoid swelling: edema of connective tissue; they undergo swelling, fraying, fragmentation and disintegration.

·  *Fibrinoid changes: the affected areas take on a deeply eosinophilic appearance resembling fibrin – fibrinoid degeneration or necrosis.

·  *Cellular reactions: infiltration by lymphocytes, plasma cells, histiocytes and fibroblasts. Pathognomonic focal inflammatory nodules called Aschoff bodies are the most characteristic in the heart. In its early phase, they constitute a foci of fibrinoid necrosis, initially surrounded by lymphocytes, macrophages and a few plasma cells.

· *Sclerosis: Aschoff bodies or diffuse inflammatory cellular infiltration are slowly replaced by fibrous scar mainly around the vessels.

    The microscopic findings are basically describing the Aschoff bodies and the 3rd and 4th stages of rheumatic fever.

142.

One  of the factors that  cause obesity is the  inhibition  of  fatty  acids  oxidation due to:

Explanation

image

The major pathway for catabolism of saturated fatty acids is a mitochondrial pathway called β-oxidation. After a long-chain fatty acid (LCFA) enters a cell, it is converted in the cytosol to its Co-A derivative. Because β-oxidation occurs in the mitochondrial matrix, the fatty acid must be transported across inner mitochondrial membrane which is impermeable to Co-A. therefore, a specialized carrier transports the long chain acyl group from the cytosol into the mitochondrial matrix. This carrier is carnitine and this rate-limiting transport process is called the carnitine shuttle. Since carnitine helps the mitochondria utilize energy, it plays a critical role in reducing the occurrence and impact of obesity. In addition to helping the mitochondria burn fat as energy, carnitine is also vital for removing waste products from mitochondria. Obesity and aging contribute to low carnitine levels, which compromises mitochondrial performance and increases insulin resistance, promoting further obesity and carnitine reduction.

143.

The genetic defect of pyruvate carboxylase deficiency is the cause of delayed physical and mental development and  early  death  in children.  This  defect is characterized by lacticemia,  lactaciduria,  disorder of a number  of metabolic pathways.  In particular, the following process is inhibited:

Explanation

In mitochondria, pyruvate carboxylase catalyzes the conversion of pyruvate to oxaloacetate (in gluconeogenesis). This enzyme requires biotin and ATP. Oxaloacetate can replenish citric acid cycle (CAC) or be used in gluconeogenesis. So, a deficiency of pyruvate decarboxylase impairs both CAC and gluconeogenesis.

144.

Deficiency  of  linoleic  and  linolenic acids   in   the   body   leads   to   the   skin damage,  hair  loss, delayed  wound  healing,  thrombocytopenia, low  resistance to infections.  These  changes  are  most likely to be caused by the impaired synthesis of the following substances:

Explanation

Two fatty acids are dietary essentials in humans – linoleic acid and α-linolenic acid (long chain polyunsaturated fatty acids). They are required for fluidity of membrane structure and synthesis of eicosanoids. A deficiency of essential fatty acid is characterized by scaly dermatitis, hair loss and poor wound healing. Essential fatty acid deficiency however is rare. Linoleic acid is the precursor of arachidonic acid, the substrate for prostaglandin synthesis. Arachidonic acid becomes essential if linoleic acid is deficient in the diet.

145.

During  ventricular systole, the cardiac muscle does not respond to additional stimulation because  it is in the phase of:

Explanation

krushkrok No145 (2013)

Absolute refractory period is the period during which the muscle does not show any response at all, whatever may be the strength of the stimulus. It is because the depolarization occurs during this period. So, a second depolarization is not possible in cardiac muscle, absolute refractory period extends throughout systole (contraction) period – 0.27sec. And, relative refractory period extends during first half of diastole (relaxation) period – about 0.26sec. Relative refractory period is the period during which the muscle shows some response if the strength of stimulus is increased to maximum.

146. A mother had  taken  synthetic hormones  during   pregnancy.   Her daughter was born with hirsutism formally resembling  of adrenal syndrome.  Such manifestation of variability is called:

Explanation

Phenocopy is an environmentally induced phenotype mimicking one usually produced by a specific genotype. An individual exhibiting such a phenotype; the simulated trait is a phenocopy. The phenotype hirsuitism mimicks adrenal syndrome.

147. Since a patient has had myocardial infarction, atria and ventricles contract independently from each other  with a frequency of 60-70 and 35-40 per minute. Specify the type of heart block in this case:

Explanation

Complete atrioventricular block (third degree heart block; complete heart block) is the condition in which the impulses produced by sinoatrial node (SA node) cannot reach the ventricles. Because of this, the ventricles beat in their own rhythm, independent of atrial beat. Partial or 2nd degree heart block: transmission of impulses produced from SA node fail to reach the ventricles. It iis slowed down and not blocked completely. Sinoatrial block is failure of impulse transmission from SA node to atrioventricular (AV) node. Intra-atrial block: transmission of impulse through atrial myocardium is impaired. Intraventricular block: conduction of impulse through His bundle and its branches is impaired.

148.

A 67-year-old male patient consumes eggs,  pork   fat,  butter,  milk  and   meat. Blood   test   results:   cholesterol  -   12,3 mmol/l, total lipids - 8,2 g/l, increased  low- density lipoprotein fraction  (LDL). What type of hyperlipoproteinemia is observed in the patient?

Explanation

Type IIa (familial hyperlipoproteinemia: ↑LDL and cholesterol. Autosomal dominant; due to absent or defective LDL receptors.

Type I:  ↑chylomicrons, triacylglycerol (TAG), cholesterol. Autosomal recessive; due to lipoprotein lipase deficiency or altered apolipoprotein C-II.

Type IV: ↑very low density lipoprotein (VLDL) and TAG. Autosomal dominant; due to hepatic overproduction of VLDL.

 

Type IIb: similar to Type IIa, except that VLDL is also increased and VLDL is normal for IIa.

149.

A 12-year-old child has a viral infection  complicated  by  obstructive  bronchitis. Bronchospasm can  be  eliminated by inhalations of a drug  from  the  following pharmacological group:

Explanation

β2-receptors on the bronchial smooth muscle cause bronchodilation which can relieve the bronchospasm (sympathetic effect). β2 adrenergic blockers will increase the bronchospasm and make the patients condition worst.  N-cholinomimetics will have similar effects as β2 adrenergic blockers. M-anticholinergics (e.g. scopolamine) have dilatory effects but β2 agonists (e.g. salbutamol, fenoterol, salmoterol) remains the best drugs for relieving bronchoconstriction (bronchospasm) without significant cardiac stimulation unlike M-cholinoblocker which might likely increase cardiac activity by inhibiting vagus effect on heart.

150.

In course of an experiment there  has been  an  increase  in the  nerve  conduction velocity. This may be caused by an increase in the concentration of the following ions that are present in the solution around the cell:

Explanation

Nerve signals are transmitted by action potential (AP), which are rapid changes in the membrane potential that spread rapidly along the nerve fiber membrane. During the depolarization stage of an AP, the membrane suddenly becomes permeable to sodium ion (Na+), allowing tremendous number of positively charged Na+ to diffuse to the interior of the axon. A major function of voltage-gated calcium ion (Ca2+) channels is to contribute to the  depolarizing phase on the action potential in some cells. Although the gating of calcium channel is slow (slow channels), in contrast to the fast sodium channels. Therefore, sodium channels play a key role in initiation and conduction of action potentials.

151.

A male working  as a blacksmith  has been  tested  for auditory acuity. The  tests revealed 50% hearing loss in the low-frequency range  and a near-normal auditory  acuity  in the  high-frequency range. This condition has been caused by the damage  to the following structures of the auditory system:

Explanation

FullSizeRender (26)

FullSizeRender (25)

Two membranes divide the spiral canal of cochlea into 3 compartments – vestibular and basilar membrane. Along the basilar membrane are 20,000-30,000 tiny fibers called basilar fibers. Each fiber has different size and shape. Fibers near the oval window (base) are short and stiff (narrowest), while approaching towards helicotrema (apex), the basilar fibers gradually become longer and soft (widest). The motion of the basilar membrane is generally described as a travelling wave. The parameters of the membrane (stiffness, narrow, wide) at a given point along its length determine its characteristic frequency at which it is most sensitive to sound vibrations. High frequency sounds localize near the base of the cochlea (near the round and oval windows), while low frequency sounds localize near the apex (helicotrema); and the middle frequency sounds – middle part of helix.

There is a 50% hearing loss in the low frequency range; that means there is damage near the apex (helicotrema).

152.

In   our   country,   routine  preventive  vaccinations against  poliomyelitis involve  using  live vaccine  that  is administered  orally.  What  immunoglobulins are responsible for the  development of local post-vaccination immunity in this case?

Explanation

Vaccine is a substance that is introduced into the body to prevent the disease produced by certain pathogens. Vaccine consists of dead pathogens or live but attenuated (artificially weakened) organisms. The vaccine induces immunity against the pathogen, either by production of antibodies or by activation of T-lymphocytes. Mature B cells express IgM and IgD on their surfaces. They may differentiate in germinal centers of lymph nodes by isotype switching into plasma cells that secrete IgA, IgE, IgG. IgA prevents attachment of bacteria and viruses to mucous membranes; it has 2 isotypes – a monomer (in circulation) and a dimer (when secreted- secretory). Crosses epithelial cells by transcytosis. It is the most produced antibody overall, but has lower serum concentrations. Released into secretions (tears, saliva, mucus) and breast milk. Based on the localization of secretory IgA, it is responsible for local immunity. And all other Immunoglobulins, since they are found in serum, they are responsible for systemic immunity.

153.

An experiment proved  that UV-irradiated skin cells of patients with xeroderma pigmentosum restore the native structure of DNA  slower than the cells of  healthy   people   due  to  the  defect  in repair  enzyme.  What  enzyme  takes  part in this process?

Explanation

Exposure of a cell to ultraviolet (UV) light can result in the covalent joining of two adjacent pyrimidines (usually thymine), producing a dimer. These thymine dimers prevent DNA polymerase from replicating the DNA strand beyond the site of dimer formation. First, a UV-specific endonuclease recognizes the dimer an cleaves the damaged strand on both the 5’-side and 3’-side of the dimer. Pyrimidine dimers can be formed in the skin cells of humans exposed to unfiltered sunlight. In the rare genetic disease Xeroderma pigmentosum, the cells cannot repair the damaged DNA because UV-specific endonuclease is defective, resulting in extensive accumulation of mutations and consequently, skin cancers.

154.

A  patient  who  has  recently   come from   an   endemic    area   presents  with elevated body temperature, headache, chills, malaise,  that  is with the symptoms which  are   typical  for  a  common   cold. What laboratory tests are necessary to confirm   or   to   refute   the   diagnosis   of malaria?

Explanation

For malaria diagnosis, microscopy of blood smears, utilizing blood films; it is the most economic, preferred and reliable diagnosis of malaria because each of the 4 major parasite species has distinguishing characteristics (Plasmodium vivax, ovale, falciparum and malariae).

155.

What  condition may  develop  15-30 minutes after re-administration of the antigen as a result of the increased level of antibodies, mainly IgE, that are adsorbed on the  surface  of target  cells, namely  tissue basophils (mast cells) and blood basophils?

Explanation

FullSizeRender (17)

Type I Hypersensitivity reaction (HSR); anaphylactic and atopic: free antigen cross-links IgE on presensitized (i.e. exposed to the antigen before) mast cells and basophils, triggering immediate release of vasoactive amines that act at postcapillary venules (i.e. histamine). Reaction develops rapidly after antigen exposure because of preformed antibody from first exposure. IgE is the main immunoglobulin involved in type I HSR. Type I: uses IgE and IgG4

Type I (Immediate, Anaphylaxis, Reagin): IgE (immunoglobulin E)-dependent activation of mast cells/basophils, usually accompanied by eosinophilia e.g. urticaria (hives), hay fever, asthma (wheezing), rhinitis and conjunctivitis (stuffy nose and itchy eyes; usually seasonal)

Type II (cytotoxic): antibody dependent reactions e.g. Goodpasture syndrome, Myasthenia gravis, Graves disease, ABO hemolytic disease of newborn etc.

Type III (immune-complex): deposition of antigen-antibody complexes e.g. systemic lupus erythromatous (SLE), Arthus reaction, serum sickness, poststreptococcal glomerulonephritis etc.

Type IV (cell mediated, delayed): antibody-independent T-cell mediated reactions e.g. positive mantoux reaction (tuberculin test), hashimoto’s thyroiditis or transplant rejection etc.

156.

10 days  after  having  quinsy  caused by   beta-hemolytic  streptococcus   a   6- year-old child exhibited symptoms of glomerulonephritis. What  mechanism   of glomerular lesion  is  most  likely  in  this case?

Explanation

FullSizeRender (17)

Peritonsillar abscess also known as quinsy ( a complication of tonsillitis) caused by both aerobic and anaerobic bacteria (streptococcus, staphylococcus and haemophilus).

Acute poststreptococcal glomerulonephritis: most frequently seen in children. Occurs approximately 2 weeks after group A streptococcal infection of pharynx or skin. Resolves spontaneously. Type III hypersensitivity reaction (Immune complex). Presents with peripheral and periorbital edema, cola-coloured urine, hypertension. On immunofluorescent microscopy: granular appearance due to IgG, IgM and C3 deposition along glomerular basement membrane and mesangium. On electron microscopy: subepithelial immune complex humps. On light microscopy: glomeruli enlarged and hypercellular.
157.

A   22-year-old    woman   ate   some seafood.  5 hours  later  the  trunk  and  the distal  parts   of  limbs  got  covered   with small itchy  papules  which  were  partially fused  together. After   one  day,  the  rash disappeared spontaneously. Specify the hypersensitivity mechanism  underlying these changes:

Explanation

It is anaphylactic because of the relatively short time it took to manifest the symptoms. It is local because, it affects just the trunk and distal part of limb. Systemic will have systemic effects like anxiety, loss of consciousness, shock, difficulty in breathing.

Type I Hypersensitivity reaction (HSR); anaphylactic and atopic: free antigen cross-links IgE on presensitized (i.e. exposed to the antigen before) mast cells and basophils, triggering immediate release of vasoactive amines that act at postcapillary venules (i.e. histamine). Reaction develops rapidly after antigen exposure because of preformed antibody from first exposure. IgE is the main immunoglobulin involved in type I HSR. Type I: uses IgE and IgG4

158.

A hypertensive patient had been keeping    to   a   salt-free   diet   and   taking  antihypertensive drugs  together with hydrochlorothiazide for a long time. This resulted in  electrolyte imbalance.   What disorder  of the internal environment occurred in the patient?

Explanation

Hydrochlorothiazide is a thiazide diuretic which acts mainly in the distal convoluted tubule to decrease the reabsorption of Na+, apparently by inhibition of a Na+/Cl- co-transporter on the luminal surface of the tubules. Therefore, thiazide diuretics cause diuresis with increase Na+ and Cl-  excretion; Hypochloremic alkalosis results from either low chloride intake or excessive chloride excretion (thiazide diuretics). Low chloride intake (salt-free diet) is very uncommon, excessive chloride excretion often occurs in hospitalized children, usually as a result of diuretic therapy (e.g. thiazide diuretic) or nasogastric tube suctioning.

159.

A miner consulted  a physician about the  appearance of body rash followed  by a loss of appetite, bloating, duodenal pain, frequent bowel movements, dizziness. Ovoscopic  probes  of feces and  duodenal contents revealed  some eggs covered with a  transparent  membrane through which 4-8 germinal  cells  could  be  seen.  What disease  is likely  to  have  occurred in the patient?

Explanation

Ancylostomiasis also known as Miner’s anemia is caused by Ancylostoma (hookworm). Larvae penetrate the body and produces an intestinal infection causing anemia by sucking blood from intestinal walls. Findngs: intestinal bleeding, abdominal pains, anemia, severe diarrhea and malnutrition. This is consistent with the symptoms in the question.

160. Children with Lesch-Nyhan syndrome have a severe form of hyperuricemia  accompanied by the  formation of tophi,  urate  calculi  in the  urinary  tracts, as well as serious neuro-psychiatric disorders.  The  cause  of this  disease  is the reduced activity of the following enzyme:

Explanation

image

Lesch-Nyhan syndrome: defective purine salvage due to absent hypoxanthine-guanine phosphoribosyl transferase (HGPRT), which converts hypoxanthine to inosine monophosphate (IMP) and guanine to Guanosine monophosphate (GMP). Results in excess uric acid production and de novo purine synthesis. This syndrome is X-linked recessive. Findings: intellectual disability, self-mutilation, aggression, hyperuricemia, gout, dystonia. It can be treated with allopurinol and febuxostat.

161.

In a car accident  a man  got injured and  lost a lot of blood.  What  changes  in peripheral blood  are most likely to occur on the 2nd day after the injury?

Explanation

Pathogenesis of acute posthemorrhagic anemia consists of 3 periods: reflex vascular period; hydremic period (2-3days) and regenerative (bone-marrow) period. During the hydremic period after 2-3days: there is compensation of circulating plasma volume: diminishing/decrease of excretion by kidneys; increase transport of interstitial fluid and lymph into the lumen of blood vessels. This will cause a relative decrease in erythrocytes (erythropenia) and hemoglobin [but colour index (C.I.) is near norm] because the fluid component of blood have been replaced by the 2nd day but the cellular components are yet to be replaced because erythropoiesis will take a longer time.

162.

In   the   surgical   ward,   the   dressing  material  was  undergoing  sterilization  in  an  autoclave.   Through   an  oversight  of a nurse  the  mode  of sterilization was changed  and  the  temperature in the autoclave  reached only 100oC instead  of the due 120oC . What microorganisms can stay viable under  these conditions?

Explanation

Some bacteria can form spores at the end of the stationary phase when nutrients are limited. Spores are highly resistant to heat and chemicals. Spores have diplocinic acid in their core; have no metabolic activity. To kill spores, we must autoclave (as is done to surgical instruments) by steaming at 121oc for 15 minutes. Examples of species that form spores and disease caused is Clostridum botulinum – Botulism; Bacillus anthracis – Anthrax; Clostridium tetani – Tetanus; Coxiella burnetti – Q fever. Amongst the two groups of spore forming microorganisms, Clostridium is more heat resistant than Bacillus. Temperatures of 110oc will kill most Bacillus spores within a short time. In the case of Clostridium, temperatures of up to 121o c are needed to kill the spores within a relatively short time.

163.

As  a  result  of  a  mechanical   injury an  over  10  cm  long  portion  of  a  peripheral nerve was damaged.  This led to the impairment of the upper limb activity. The patient was  offered  nerve  transplantation. What glial cells will participate in regeneration and provide  the trophism  of the injured limb?

Explanation

Schwann cells are the major glial cells in peripheral nervous system (PNS). They provide myelination (insulation); also promote axonal regeneration. Derived from neural crest. Increase conduction velocity via salutatory conduction at the nodes of ranvier, where there is a high concentration of Na+ channels. They may be injured in Guillain-Barrė syndrome. Fibrous cells and protoplasmic cells are types of astrocytes forming blood-brain barrier, supporting network, mainataining the chemical environment and involved in recycling of neurotrtansmitters. Microglial cells are derived from monocytes – they engulf and destroy microorganisms and cellular debris. Ependymal cells form the epithelial lining of the ventricles of the brain and spinal cord.

164. A  26-year-old  woman  at  40 weeks pregnant has been  delivered  to the maternity ward.  Objectively:  the  uterine cervix is opened, but the contractions are absent.  The  doctor  has administered her a hormonal drug  to  stimulate the  labor. Name this drug:

Explanation

Oxytocin is a hormonal drug inducing rhythmic uterine contractions by acting on myometrium cell membranes, increasing their excitability. Oxytocin remains the strongest stimulator of uterine contractions. Estrone is a female sex hormone, but small amounts are also produced in adrenal cortex in both sexes. Its effects on labour is probably to stimulate cervical ripening through prostaglandin synthesis and not on uterine contraction. The other options are also related to the adrenal gland but no direct effect on uterine contraction.

165.

A  patient has  recurrent attacks   of epileptic seizures and stays unconscious between them.  In order  to stop  convulsions   the   drugs   of  the   following   group should be used in the first place:

Explanation

Tranquilizers (anxiolytics) are drugs which reduce state of internal tension, morbid phobia and anxiety, they are used to treat neurosis. Because many tranquilizers also cause some sedation, they can function as both anxiolytic and hypnotic (sleep-inducing) agents, some also have anticonvulsant activity (epileptic seizures). Analeptics (CNS, respiratory stimulants); Neuroleptics (antischizophrenic, antipsychotic); Sedatives (decrease excitement without inducing sleep).

166.

A patient with arthritis and varicose veins has been taking a non-steroidal anti-inflammatory drug for a long time, which caused  thrombosis of  skin  veins.  Which of the following drugs might have caused this complication?

Explanation

Celecoxib is a selective cyclooxygenase 2 (COX-2) inhibitor. Approved for treatment of rheumatoid arthritis, osteoarthritis, acute to moderate pain, also approved for patients with peptic ulcers or gastroduodenitis. Selective COX-2 inhibitors lack disaggregative properties (i.e. they can’t prevent platelet aggregation), unlike non-selective inhibitors of COX-1 and COX-2. Therefore, there is probability of development of thrombosis when used alone. All other options are non-selective inhibitors of COX-1 and COX-2; which means they have disaggregating properties (especially aspirin).

167.

Students  study the stages of gametogenesis. They  analyze  a cell having  a  haploid   number   of  chromosomes, and each chromosome consists of two chromatids. The chromosomes are located in the equatorial plane of the cell. Such situation is typical for the following stage of meiosis:

Explanation

Gametogenesis involves the generation of germ cells (gametes) through meiosis. Meiosis requires 2 cell divisions, meiosis I and II, in order to reduce the number of chromosomes to the haploid number of 23. At the beginning of meiosis I, germ cells replicate their DNA so that each of the 46 chromosomes is duplicated into sister chromatids. Homologous pairs then separate into two daughter cells, thereby reducing the chromosome number from diploid to haploid. Shortly thereafter, meiosis II separates sister chromatids (2 sister chromatids are joined in the middle by a centromere). Each gamete then conatins 23 chromosomes.

 

Metaphaseis a stage of cell division in which chromosomes are at their most condensed and coiled stage and chromosomes are arranged along the equatorial plane during this phase. Cell division is usually studied during this phase.

168.

A  35-year-old   female  patient underwent biopsy  of the  breast  nodules. Histological  examination revealed enhanced proliferation of the  small duct epithelial  cells  and   acini,  accompanied by the  formation of glandular structures of various  shapes  and  sizes, which  were located  in the fibrous stroma.  What is the most likely diagnosis?

Explanation

FullSizeRender (21)

Adenoma: benign epithelial tumor from the epithelium of the glands and glandular organs. More often they can be found in the breast, thyroid gland, liver, ovaries, prostate gland, GIT. In the question, it is located in the fibrous stroma – FIBROADENOMA.

169. Examination of the duodenal contents revealed  some pear-shaped protozoa with  two  nuclei  and  four  pairs  of  flagella.  The   organisms   had   also two  axostyles  between the  nuclei  and  a ventral adhesive disc. What protozoan representative was found in the patient?

Explanation

image

image

Giardia lamblia is pear shape or “clownface” with two nuclei, 4 pairs of flagella. The two nuclei are outlined by adhesive discs. Transmitted by cysts in water. The cysts are oval, have 4 nuclei and have clearly visible axostyles. It causes Giardiasis – bloating, flatulence, foul-smelling, fatty diarrhea. Diagnosis: trophozoites or cysts in stool.

170.

A   specimen   of  an   onion   rootlet includes  a  cell  in  which  the  fully condensed  chromosomes are  located   in the equatorial plane making the monaster. What phase of the mitotic cycle is the cell in?

Explanation

krushkrok No170 (2013)

Gametogenesis involves the generation of germ cells (gametes) through meiosis. Meiosis requires 2 cell divisions, meiosis I and II, in order to reduce the number of chromosomes to the haploid number of 23. At the beginning of meiosis I, germ cells replicate their DNA so that each of the 46 chromosomes is duplicated into sister chromatids. Homologous pairs then separate into two daughter cells, thereby reducing the chromosome number from diploid to haploid. Shortly thereafter, meiosis II separates sister chromatids (2 sister chromatids are joined in the middle by a centromere). Each gamete then conatins 23 chromosomes.

Metaphase is a stage of cell division in which chromosomes are at their most condensed and coiled stage and chromosomes are arranged along the equatorial plane during this phase. Cell division is usually studied during this phase.

171.

When examining a patient, the doctor revealed  a tumor  of the  bronchus  which borders  on the  aorta.  Which  bronchus  is affected?

Explanation

The bronchi form the bronchial tree that consists of main bronchi, lobar bronchi, segmental bronchi and their branches and terminal bronchioles. We have 2 main/primary/principal bronchi: left and right. Left main bronchus (bronchus principalis sinister) is approximately 4-5cm long; is longer and narrower than the right main bronchus. It runs slantwise leftwards to enter the hilum of left lung. The left main bronchus neighbors the arch of aorta anteriorly and the descending aorta and esophagus posteriorly. Right main bronchus neighbors the azygos vein, which runs crosswise. All other branches of the bronchi tree are found in the lungs.

172.

A 54-year-old  female was brought to the  casualty  department after  a car accident. A traumatologist diagnosed her with multiple  fractures of the lower extremities. What  kind of embolism  is most likely to develop in this case?

Explanation

Embolism is occlusion of a vessel by material travelling in the circulation.

* Fat/Adipose embolism: obstruction of arterioles and capillaries by fat globules constitutes fat embolism. It may occur following severe fracture trauma to bones, inflammation of bones and soft tissues, fatty liver

* Thromboembolism: a detached thrombus or part of thrombus which may arise in the arterial or venous circulation.

* Gas embolism: two main forms of gas embolism are air embolism and decompression sickness. Air embolism is usually due to accidental pumping of air into the venous circulation during intravenous (IV) injection or transfusion ( bubble – air escaped).

 

Tissue embolism: fragments of tissue.

173. Microscopy     of     the     bronchial wall revealed atrophy  of the mucosa, metaplastic change from columnar  to squamous  epithelium,   an   increase    in the number of goblet cells, diffuse infiltration    of   the   bronchial  wall   with     lymphoplasmacytic    elements    with a large number of neutrophilic granulocytes, pronounced sclerosis. Specify the morphological form of bronchitis:

Explanation

Pronounced sclerosis indicates a chronic condition. The diffuse infiltration of bronchial wall with lymphoplasmocytic elements with a large number of neutrophils indicates a purulent inflammation. Bronchial wall – bronchitis

174.

Due  to the blockage  of the common bile duct (which was radiographically confirmed),  the  biliary  flow to  the duodenum was stopped. We should expect the impairment of:

Explanation

Obturation (obstruction, to close) of bile duct – it can be:

* Intrahepatic – blockage of intrahepatic bile ducts

* Extrahepatic – blockage of common bile duct (ductus choledochus).

Findings:

* malabsorption: bile salts do not enter the Small Intestine; no emulsification of fat.

*light coloured stool: due to lack of urobilin (which leads to lack of stercobilin).

*Jaundice (posthepatic, mechanic, obstructive): increased conjugated Bilirubin.

* Steatorrhea

 

The findings are specific for obstruction of bile duct and bile acid deficiency.

175.

Typical  manifestations of  food  poisoning    caused    by   C.   botulinum   are double   vision,  abnormal  functioning  of the swallowing and breathing. These symptoms develop  as a result of:

Explanation

Clostridium botulinum is a gram positive rod (bacillus). They secrete botulinum toxin (an exotoxin). Findings: flaccid paralysis, floppy baby; toxin prevents release of stimulatory (acetylcholine) signals at neuromuscular junctions → flaccid paralysis. Exotoxin is secreted from certain species of gram positive and gram negative bacteria. Its toxoids are used as vaccines. Inhibition of the release of stimulatory neurotransmitters can lead to the symptoms listed in the question.

176.

At  the  stage  of  translation  in  the rough  endoplasmic reticulum,  the ribosome  moves along the mRNA. Amino acids are joined together by peptide bonds in a specific sequence, and thus polypeptide synthesis takes place. The sequence of amino acids in a polypeptide corresponds to the sequence  of:

Explanation

The pathway of protein synthesis translates the 3-letter alphabet of nucleotide sequences on mRNA into the twenty-letter alphabet of amino acids that constitute proteins. mRNA convey genetic information from DNA to the ribosome, where they specify the amino acid sequence of the protein products of gene expression. As in DNA, mRNA genetic information is in the sequence of nucleotides, which are arranged into codons consisting of 3 bases each. Each codon encodes for a specific amino acid, except the stop codons, which terminate protein synthesis. The mRNA is translated from its 5’-end to its 3’-end, producing a protein synthesized from its amino-terminal end to its carboxyl-terminal end.

177. After  the prolonged vomiting a pregnant 26-year-old  woman  was  found to have  the  reduced volume  of circulating blood.  What  change  in the total  blood volume can be the case?

Explanation

Polycythemic hypovolemia is a condition with a decreased amount of plasma usually characterized by appreciable concentration and increased viscosity of blood. Hematocrit is above normal. This condition is observed in connection with considerable loss of water by the organism as in diarrhea, intractable vomiting, extensive burns. Hematocrit is increased because there is no loss of erythrocytes just fluid was lost from vomiting. It can’t  be hypervolemia because there is loss of fluid from prolonged vomiting.

 

In simple hypovolemia: hematocrit is normal; oligocythemic hypovolemia: hematocrit is decreased.

178.

On allergological  examination a patient has been diagnosed with pollinosis. Specific desensitization can be performed by:

Explanation

Desensitization therapy in atopic individuals involves repeated injection of increasingly greater amounts of allergen, resulting in production of IgG antibodies that attach to allergens and prevent them from binding to mast cells.

179.

A   patient   consulted    a   physician about chest pain, cough, fever. Roentgenography of lungs revealed  eosinophilic infiltrates which were found to contain  the  larvae.  What  kind  of helminthiasis   are   these   presentations  typical for?

Explanation

Ascaris lumbricoides (giant roundworm) is the causative agent of Ascariasis. Transmission: fecal-oral; eggs are visible in faeces under microscope. Some patients may have pulmonary symptoms or neurological disorders during migration of the larvae. A bolus of worms may obstruct the intestine; migrating larvae may cause pneumonitis and eosinophilia.

180.

A patient with signs of osteoporosis and urolithiasis has been admitted to the endocrinology department. Blood test revealed  hypercalcemia and hypophosphatemia. These changes are associated  with abnormal synthesis of the following hormone:

Explanation

image

Parathyroid hormone: secreted by chief cells of parathyroid gland. Effects include:

↑bone resorption of Ca2+ and PO43- → ↑their plasma levels

↑kidney reabsorption of Ca2+ in distal convoluted tubule → ↑ Ca2+ plasma level

↓reabsorption of PO43- in proximal convoluted tubule → ↓ PO43- plasma levels

↑Calcitriol (vit D3) production by stimulating kidney 1α-hydroxylase in proximal convoluted tubule. It increases Ca2+ and PO43- absorption in the intestine.

In general, parathyroid hormone ↑ Ca2+ plasma level but ↓ PO43- plasma levels. Abnormal synthesis (↑synthesis) of parathyroid hormone can lead to hypercalcemia and hypophosphatemia.

Calcitonin is secreted by parafollicular cells (C cells of the thyroid gland). It ↓bone resorption of Ca2+. It opposes actions of parathyroid hormone. But its not important in normal Ca2+ homeostasis. Calcitriol ↑ circulating Ca2+ ions as a means of enhancing intestinal absorption of calcium (NB: Calcitriol production is dependent on parathyroid hormone). Aldosterone: reabsorption of Na+, excretion of K+. Cortisol – glucocorticoid: ↑ blood pressure and gluconeogenesis; ↓ inflammatory and immune responses; ↓ bone formation (↓osteoblast activity).

181.

Histological  examination of  the  biopsy  material obtained  from  the  lower third  of  the  esophagus of  a  57-year-old male  with  the  symptoms   of  continuous reflux  revealed  the  change  of the  stratified  squamous epithelium to  the  single-layer  columnar glandular epithelium with  signs  of  mucus   production.  Specify the  pathological process  in  the  mucous membrane:

Explanation

IMG_9796

Metaplasia is a reversible change of one type to another type of adult epithelial or mesenchymal cells, usually in response to abnormal stimuli and often reverts back to normal on removal of stimulus. Types (2):

·        Epithelial: *Squamous metaplasia: in bronchus (chronic smokers), cervix.

                         *Columnar metaplasia: there is transformation to columnar epithelium.

·        Mesenchymal metaplasia: osseous and cartilaginous.

Hyperplasia: ↑number of cells; Hypertrophy: ↑in size.

182.

A  30-year-old  female  exhibits  signs of virilism (growth  of body  hair,  balding temples, menstrual disorders). This condition can be caused by the overproduction of the following hormone:

Explanation

This symptoms indicates an increase in male sex hormones. This can probably be a pathology of the adrenals or ovaries in a female, which leads to increase production of testosterone (the only male sex hormone in the options)

183. A patient with bacterial periodontitis has  been  administered iontophoresis with thuse of iodine  solution.  Specify the mechanism  of  therapeutic action  othis agent:

Explanation

184.

A  patient with  extensive  myocardial infarction  has developed heart  failure. What pathogenetic mechanism  contributed  to the development of heart  failure in the patient?

Explanation

Extensive myocardial infarction: extensive necrosis of myocardial cells. The pathogenesis of the heart failure is definitely the fact that there are less active/alive myocardial cells that can contract and pump blood effectively. No information about blood volume, so we can’t talk about the other options.

185.

A patient who had been continuously taking  drugs  blocking  the  production of angiotensin II developed bradycardia and arrhythmia. A likely cause of these disorders is:

Explanation

Angiotensin converting enzyme (ACE) inhibitors block the production of angiotensin II from angiotensin I.

 

The side effect of this group of drug is dry cough, rash, fever, hypotension, hyperkalemia. Hyperkalemia can cause bradycardia and arrhythmia.

186.

A patient has arterial hypertension. What   long-acting   drug  from  the  group of calcium channel blockers should be prescribed?

Explanation

Amlodipine is a calcium channel blocker (long acting); from the dihydropyridine family.

 

Atenolol is a β-blocker (cardioselective); Reserpine and Octadine are sympatholytics; Pyrroxanum is an adrenergic antagonist/α-adrenoblocker (non-selective α1 and α2).

187.

Human skin has a high breaking strength. It is known  that  the  skin consists of  epithelial tissue  and  two  kinds  of connective tissue. Which of the following tissues provides the skin strength?

Explanation

Connective tissue proper are divided into 2 general subtypes: loose connective tissue (LCT) and dense connective tissue (DCT). DCT is further subcategorized into: Dense irregular connective tissue and dense regular connective tissue. Dense regular connective tissue is the main functional component of tendons, ligaments and aponeuroses. Dense irregular connective tissue is characterized by abundant fibers and few cells. It contains mostly collagen fibers. Because of its high proportion of collagen fibers, it provides significant strength. Typically, its fibers are arranged in bundles oriented in various directions (thus the term “irregular”) that can withstand stresses on organs or structures. Skin contains a relatively thick layer of dense irregular connective tissue in the dermis, called the reticular or deep layer of the dermis. It provides resistance to tearing as a consequence of stretching forces from different directions.

188.

As a result of a home injury, a patient suffered a significant blood loss, which led to  a fall in blood  pressure. Rapid  blood pressure recovery  after  the  blood  loss is provided  by the following hormones:

Explanation

Adrenaline is an adrenomimetic; α1 – mediates peripheral vasoconstriction →↑blood pressure. Activation of β1-adrenoreceptors by adrenaline results in increased cardiac output and heart rate.

Vasopressin → ↓diuresis; ↑H2O reabsorption → ↑blood volume

 

Aldosterone is a steroid hormone and cannot produce the same level of increase in blood pressure as adrenaline and vasopressin combined.

189.

A  patient with  constant headaches, pain in the occipital region, tinnitus, dizziness has been  admitted to the cardiology department. Objectively: AP- 180/110 mm Hg, heart  rate - 95/min. Radiographically, there   is  a  stenosis  of  one  of  the  renal arteries. Hypertensive condition in this patient has been  caused by the activation of the following system:

Explanation

FullSizeRender (12) The main stimulator of renin release from juxtaglomerular cell is decrease blood flow in afferent arteriole. Stenosis of renal artery → ↓blood flow to afferent arteriole thereby stimulating renin release and set up the renin-angiotensin-aldosterone system (RAAS).
190.

A   patient  complains   that   at   the bare   mention  of  the  tragic  events   that once  occurred in his life he  experiences tachycardia, dyspnea  and  an  abrupt rise in blood  pressure. What  structures of the CNS are responsible for these cardiorespiratory  reactions in this patient?

Explanation

Cerebral cortex is responsible for the memory, attention, consciousness and all higher nervous system functions. The cardiorespiratory reaction in the patient is because he is able to store and remember the tragic event. The cerebral cortex also has extensive connections with the subcortical structures responsible for short and long term memories.

191.

A patient consulted  a dentist  about limited  mouth  opening  (trismus).  He has a history of a stab wound of the lower extremity. What infection may cause these symptoms?

Explanation

Tetanus is caused by Clostridium tetani that secretes a toxin called tetanospasmin. Findings: spasticity, risus sardonicus (raised eyebrows and open grin) and lockjaw (trismus); toxin prevents release of inhibitory (GABA and glycine) neurotransmitters from Renshaw cells in spinal cord. It can be prevented with tetanus vaccine.

192. Anatomical dead space is the portion of the  air that  is left in the  airways  after expiration. The reduction of the anatomical dead space is typical for the following situation:

Explanation

Anatomical dead space extends from nose up to terminal bronchiole. It includes nose, pharynx, trachea, bronchi and branches of bronchi up to terminal bronchioles. These structures serve only as the passage for air movement. Gaseous exchange does not take place in these structures.

 

Tracheostomy is a surgical procedure that creates an opening through the neck into the trachea (wind pipe). A tube is usually placed through this opening to provide an airway and to remove secretions from the lungs. This tube is called a tracheostomy tube. This procedure is usually performed when there is a large object blocking the airway; inability to breathe on your own; severe neck or mouth injuries etc. This reduces the anatomical dead space because it cuts out the nose and pharynx from the anatomical dead space.

193.

Analysis of the experimental spirogram  of a 55-year-old  person  revealed a decrease in tidal volume and respiratory amplitude compared to  the  situation   of ten  years  ago. The  change  in these  indicators is caused by:

Explanation

Tidal volume is the volume of air breathed in and out of lungs in a single normal quiet respiration. Tidal volume signifies the normal depth of breathing. This volume and respiratory amplitude is dependent primarily on the contraction and relaxation of the diaphragm (diaphragm - primary respiratory muscle). So, a decrease in this parameters equally means a decrease in force of respiratory muscle contraction.

194.

A patient underwent a course of treatment for atherosclerosis. Laboratory tests revealed an increase in the anti-atherogenic lipoprotein fraction in the blood  plasma.  The  treatment efficacy  is confirmed by the increase in:

Explanation

High density lipoprotein (HDL) – transports cholesterol from periphery to liver. Low Density Lipoprotein (LDL) - transports cholesterol from liver to tissues.

LCAT (lecithin-cholesterol acyltransferase) – catalyzes esterification of cholesterol     

Cholesterol ester transfer protein (CETP) -  mediates transfer of cholesterol esters to other lipoprotein particles.

High density lipoprotein (HDL): is the antiatherogenic lipoprotein fraction (nicknamed the ‘good lipoprotein’).

 

Low density lipoprotein (LDL): is the proatherogenic lipoprotein fraction (nicknamed the ‘bad lipoprotein’)

195.

A 65-year-old  female patient has chronic constipations due to the colon hypotonia. What  drug  should  be  chosen in this case?

Explanation

Constipations requires the use of laxatives that will stimulate the movement of food through gastrointestinal tract (GIT), produce soft formed stool and accelerate defecation. Bisacodyl is typically prescribed for relief of constipation and for the management of neurogenic bowel dysfunction as well as part of bowel preparation before medical examinations, such as for a colonoscopy. Bisacodyl works directly on the colon to produce bowel movement. It works by stimulating enteric nerves to cause colonic mass movements (contractions); removing the colon hypotonia.

196.

A   female    patient   complains    of vision    impairment.    On     examination she was found to have obesity, fasting hyperglycemia. What  complication of diabetes  can cause vision impairment?

Explanation

Microangiopathy is a chronic complication of diabetes mellitus (fasting hyperglycemia) affecting small blood vessels and manifested as retinopathy (vision impairment), nephropathy, neuropathy. Macroangiopathy affects larger vessels causing symptoms such as gangrene, ulceration, transient ischemic attack.
197.

Analysis of the family history of children with Van der Woude syndrome revealed  that  in their  families one  of the parents had the typical for this syndrome defects   (cleft   lip   and   palate,   lip   pits regardless of  gender). What  is the  type of inheritance of this syndrome?

Explanation

image

Autosomal dominant: often due to defects in structural genes. Many generations, both male and female are affected. Found in every generation; no generation is left out (skipped). Parent – child in every generation.

Autosomal recessive is usually seen in some generations (other generations are skipped).

It is sex linked, if only males or only females(X-linked) are affected.

198.

Administration of doxycycline hydrochloride caused an imbalance of the symbiotic   intestinal  microflora.   Specify the kind of imbalance caused by the antibiotic therapy:

Explanation

Administration of doxycycline hydrochloride (antibiotic) can cross react with the normal microflora in the intestine thereby causing dysbacteriosis (an imbalance in the normal microflora). All broad spectrum antibiotics can be associated with dysbacteriosis.
199.

A  3-year-old  child  had  eaten  some strawberries. Soon  he  developed a  rash and itching. What was found in the child’s leukogram?

Explanation

This is an example of type I hypersensitivity reaction considering the time taken for the symptoms (rash and itching) to manifest and the expression of eosinophilia.

Neutrophilic leukocytosis: bacterial infection

Lymphocytosis: viral and bacterial infection (e.g. whooping cough, tuberculosis)

 

Monocytosis: chronic inflammation (tuberculosis); autoimmune disease

200. A  12-year-old  patient was found  to have  blood  serum  cholesterol at the  rate of  25 mmol/l.  The  boy  has  a  history  of hereditary familial hypercholesterolemia, which is caused by the impaired  synthesis of the following protein receptors:

Explanation

Type IIa (familial hyperlipoproteinemia: ↑LDL and cholesterol. Autosomal dominant; due to absent or defective LDL receptors.

Type I:  ↑chylomicrons, triacylglycerol (TAG), cholesterol. Autosomal recessive; due to lipoprotein lipase deficiency or altered apolipoprotein C-II.

Type IV: ↑very low density lipoprotein (VLDL) and TAG. Autosomal dominant; due to hepatic overproduction of VLDL.

 

Type IIb: similar to Type IIa, except that VLDL is also increased and VLDL is normal for IIa.